Maternal Nurseslabs

You might also like

Download as pdf or txt
Download as pdf or txt
You are on page 1of 56

The nurse is counseling a couple who has sought Correct Answer: D. Obtain a serum glucose level.

information about conceiving. The couple asks the nurse to


explain when ovulation usually occurs. Which statement by 5. Question1 point(s)
the nurse is correct? Which statement made by the client indicates that the
A. Two weeks before menstruation. mother understands the limitations of breastfeeding her
newborn?
B. Immediately after menstruation.
A. "Breastfeeding my infant consistently every 3 to 4 hours
C. Immediately before menstruation. stops ovulation and my period."

D. Three weeks before menstruation. B. "Breastfeeding my baby immediately after drinking


alcohol is safer than waiting for the alcohol to clear my
Correct Answer: A. Two weeks before menstruation breast milk."
2. Question1 point(s) C. "I can start smoking cigarettes while breastfeeding
The nurse instructs a laboring client to use accelerated blow because it will not affect my breast milk."
breathing. The client begins to complain of tingling fingers D. "When I take a warm shower after I breastfeed, it
and dizziness. Which action should the nurse take? relieves the pain from being engorged between
A. Administer oxygen by face mask. breastfeedings."

B. Notify the health care provider of the client's symptoms. Correct Answer: A. “Breastfeeding my infant consistently
every 3 to 4 hours stops ovulation and my period.”
C. Have the client breathe into her cupped hands.
6. Question1 point(s)
D. Check the client's blood pressure and fetal heart rate.
When assessing the adequacy of sperm for conception to
Correct Answer: C. Have the client breathe into her cupped occur, which of the following is the most useful criterion?
hands.

3. Question1 point(s)
A. Sperm count
When assessing a client at 12 weeks of gestation, the nurse
recommends that she and her husband consider attending B. Sperm motility
childbirth preparation classes. When is the best time for the C. Sperm maturity
couple to attend these classes?
D. Semen volume
A. At 16 weeks of gestation.
Correct Answer: B. Sperm motility
B. At 20 weeks of gestation.
7. Question1 point(s)
C. At 24 weeks of gestation.
A couple who wants to conceive but has been unsuccessful
D. At 30 weeks of gestation. during the last 2 years has undergone many diagnostic
Correct Answer: D. At 30 weeks of gestation. procedures. When discussing the situation with the nurse,
one partner states, “We know several friends in our age
One hour following a normal vaginal delivery, a newborn group, and all of them have their own child already, Why
infant boy’s axillary temperature is 96° F, his lower lip is can’t we have one?”. Which of the following would be the
shaking and, when the nurse assesses for a Moro reflex, the most appropriate nursing diagnosis for this couple?
boy’s hands shake. Which intervention should the nurse
implement first? A. Fear related to the unknown.

A. Stimulate the infant to cry. B. Pain related to numerous procedures.

B. Wrap the infant in warm blankets. C. Ineffective family coping related to infertility.

C. Feed the infant formula. D. Self-esteem disturbance related to infertility.

D. Obtain a serum glucose level.


Correct Answer: D. Self-esteem disturbance related to Correct
infertility.
Correct Answer: C. Relaxation of the pelvic joints
8. Question1 point(s)
12. Question1 point(s)
Which of the following urinary symptoms does the
pregnant woman most frequently experience during the Which of the following represents the average amount of
first trimester? weight gained during pregnancy?

A. Dysuria A. 12 to 22 lb

B. Frequency B 15 to 25 lb

C. Incontinence C. 24 to 30 lb

D. Burning D. 25 to 40 lb

Correct Answer: B. Frequency Correct Answer: C. 24 to 30 lb

9. Question1 point(s) 13. Question1 point(s)

Heartburn and flatulence, common in the second trimester, When talking with a pregnant client who is experiencing
are most likely the result of which of the following? aching swollen leg veins, the nurse would explain that this
is most probably the result of which of the following?
A. Increased plasma HCG levels
A. Thrombophlebitis
B. Decreased intestinal motility
B. Pregnancy-induced hypertension
C. Decreased gastric acidity
C. Pressure on blood vessels from the enlarging uterus
D. Elevated estrogen levels
D. The force of gravity pulling down on the uterus
Correct Answer: C. Decreased gastric acidity
Correct Answer: C. Pressure on blood vessels from the
10. Question1 point(s) enlarging uterus

In which of the following areas would the nurse expect to 14. Question1 point(s)
observe chloasma?
Cervical softening and uterine souffle are classified as which
A. Breast, areola, and nipples of the following?

B. Chest, neck, arms, and legs A. Diagnostic signs

C. Abdomen, breast, and thighs B. Presumptive signs

D. Cheeks, forehead, and nose C. Probable signs

Correct Answer: D. Cheeks, forehead, and nose D. Positive signs

11. Question1 point(s) Correct

A pregnant client states that she “waddles” when she walks. Correct Answer: C. Probable signs
The nurse’s explanation is based on which of the following
is the cause? 15. Question1 point(s)

A. The large size of the newborn. Which of the following would the nurse identify as a
presumptive sign of pregnancy?
B. Pressure on the pelvic muscles.
A. Hegar sign
C. Relaxation of the pelvic joints.
B. Nausea and vomiting
D. Excessive weight gain.
C. Skin pigmentation changes
D. Positive serum pregnancy test 19. Question1 point(s)

Correct Answer: B. Nausea and vomiting Which of the following would cause a false-positive result
on a pregnancy test?
16. Question1 point(s)
A. The test was performed less than 10 days after an
Which of the following common emotional reactions to abortion.
pregnancy would the nurse expect to occur during the first
trimester? B. The test was performed too early or too late in the
pregnancy.
A. Introversion, egocentrism, narcissism.
C. The urine sample was stored too long at room
B. Awkwardness, clumsiness, and unattractiveness. temperature.
C. Anxiety, passivity, extroversion. D. A spontaneous abortion or a missed abortion is
D. Ambivalence, fear, fantasies. impending.

Correct Answer: D. Ambivalence, fear, fantasies Correct Answer: A. The test was performed less than 10
days after an abortion
17. Question1 point(s)
20. Question1 point(s)
During which of the following would the focus of classes be
mainly on physiologic changes, fetal development, FHR can be auscultated with a fetoscope as early as which
sexuality, during pregnancy, and nutrition? of the following?

A. Prepregnant period A. 5 weeks gestation

B. First trimester B. 10 weeks gestation

C. Second trimester C. 15 weeks gestation

D. Third trimester D. 20 weeks gestation

Correct Answer: B. First trimester Correct Answer: D. 20 weeks gestation

to bond with and get support from other moms-to-be. This 21. Question1 point(s)
is also the time when screening tests or amniocentesis will A client LMP began July 5, 2020. Her estimated date of
be offered, as well as other chromosomal abnormalities, delivery (EDD) should be which of the following?
genetic disorders, and neural tube defects.
A. January 2, 2021
Option D: Third-trimester classes may focus on preparation
for birth, parenting, and newborn care. B. March 28, 2021

18. Question1 point(s) C. April 12, 2021

Which of the following would be a disadvantage of D. October 12, 2020


breastfeeding?
Correct Answer: C. April 12, 2021
A. Involution occurs more rapidly.
22. Question1 point(s)
B. The incidence of allergies increases due to maternal
antibodies. Which of the following fundal heights indicates less than 12
weeks’ gestation when the date of the LMP is unknown?
C. The father may resent the infant’s demands on the
mother’s body. A. Uterus in the pelvis

D. There is a greater chance for error during preparation. B. Uterus at the xiphoid

Correct Answer: C. The father may resent the infant’s C. Uterus in the abdomen
demands on the mother’s body D. Uterus at the umbilicus
Correct Answer: A. Uterus in the pelvis D. Fourth stage

23. Question1 point(s) Correct Answer: B. Second stage

Which of the following danger signs should be reported 27. Question1 point(s)
promptly during the antepartum period?
Barbiturates are usually not given for pain relief during
A. Constipation active labor for which of the following reasons?

B. Breast tenderness A. The neonatal effects include hypotonia, hypothermia,


generalized drowsiness, and reluctance to feed for the first
C. Nasal stuffiness few days.
D. Leaking amniotic fluid B. These drugs readily cross the placental barrier, causing
Correct Answer: D. Leaking amniotic fluid depressive effects in the newborn 2 to 3 hours after
intramuscular injection.
24. Question1 point(s)
C. They rapidly transfer across the placenta, and the lack of
Which of the following prenatal laboratory test values an antagonist makes them generally inappropriate during
would the nurse consider as significant? labor.

A. Hematocrit 33.5% D. Adverse reactions may include maternal hypotension,


allergic or toxic reaction, or partial or total respiratory
B. Rubella titer less than 1:8 failure.
C. White blood cells 8,000/mm3 Correct Answer: C. They rapidly transfer across the
D. One hour glucose challenge test 110 g/dL placenta, and the lack of an antagonist makes them
generally inappropriate during labor.
Correct Answer: B. Rubella titer less than 1:8
28. Question1 point(s)
A rubella titer should be 1:8 or greater. Thus, a finding of a
titer less than 1:8 is significant, indicating that the client Which of the following nursing interventions would the
may not possess immunity to rubella. nurse perform during the third stage of labor?

25. Question1 point(s) A. Obtain a urine specimen and other laboratory tests.

Which of the following characteristics of contractions B. Assess uterine contractions every 30 minutes.
would the nurse expect to find in a client experiencing true C. Coach for effective client pushing.
labor?
D. Promote parent-newborn interaction.
A. Occurring at irregular intervals.
Correct Answer: D. Promote parent-newborn interaction.
B. Starting mainly in the abdomen.
29. Question1 point(s)
C. Gradually increasing intervals.
Which of the following actions demonstrates the nurse’s
D. Increasing intensity with walking. understanding of the newborn’s thermoregulatory ability?
Correct Answer: D. Increasing intensity with walking A. Placing the newborn under a radiant warmer.
26. Question1 point(s) B. Suctioning with a bulb syringe.
During which of the following stages of labor would the C. Obtaining an Apgar score.
nurse assess “crowning”?
D. Inspecting the newborn’s umbilical cord.
A. First stage
Correct Answer: A. Placing the newborn under a radiant
B. Second stage warmer.
C. Third stage
30. Question1 point(s) 34. Question1 point(s)

Immediately before expulsion, which of the following Which of the following is true regarding the fontanels of the
cardinal movements occur? newborn?

A. Descent A. The anterior is triangular shaped; the posterior is


diamond-shaped.
B. Flexion
B. The posterior closes at 18 months; the anterior closes at
C. Extension 8 to 12 weeks.
D. External rotation C. The anterior is large in size when compared to the
Correct Answer: D. External rotation posterior fontanel.

31. Question1 point(s) D. The anterior is bulging; the posterior appears sunken.

Before birth, which of the following structures connects the Correct Answer: C. The anterior is large in size when
right and left auricles of the heart? compared to the posterior fontanel.

A. Umbilical vein

B. Foramen ovale 35. Question1 point(s)

C. Ductus arteriosus Which of the following groups of newborn reflexes below


are present at birth and remain unchanged through
D. Ductus venosus adulthood?

Correct Answer: B. Foramen ovale A. Blink, cough, rooting, and gag

32. Question1 point(s) B. Blink, cough, sneeze, gag

Which of the following when present in the urine may cause C. Rooting, sneeze, swallowing, and cough
a reddish stain on the diaper of a newborn?
D. Stepping, blink, cough, and sneeze
A. Mucus
Correct Answer: B. Blink, cough, sneeze, gag
B. Uric acid crystals
36. Question1 point(s)
C. Bilirubin
Which of the following describes the Babinski reflex?
D. Excess iron
A. The newborn’s toes will hyperextend and fan apart from
Correct Answer: B. Uric acid crystals the dorsiflexion of the big toe when one side of the foot is
stroked upward from the ball of the heel and across the ball
33. Question1 point(s) of the foot.
When assessing the newborn’s heart rate, which of the B. The newborn abducts and flexes all extremities and may
following ranges would be considered normal if the begin to cry when exposed to sudden movement or loud
newborn were sleeping? noise.
A. 80 beats per minute C. The newborn turns the head in the direction of the
B. 100 beats per minute stimulus, opens the mouth, and begins to suck when the
cheek, lip, or corner of the mouth is touched.
C. 120 beats per minute
D. The newborn will attempt to crawl forward with both
D. 140 beats per minute arms and legs when he is placed on his abdomen on a flat
surface.
Correct Answer: B. 100 beats per minute
Correct Answer: A. The newborn’s toes will hyperextend
and fan apart from the dorsiflexion of the big toe when one
37. Question1 point(s) 40. Question1 point(s)

Which of the following statements best describes Which of the following factors would the nurse suspect as
hyperemesis gravidarum? predisposing a client to placenta previa?

A. Severe anemia leading to an electrolyte, metabolic, and A. Multiple gestation


nutritional imbalances in the absence of other medical
problems. B. Uterine anomalies

B. Severe nausea and vomiting leading to an electrolyte, C. Abdominal trauma


metabolic, and nutritional imbalances in the absence of D. Renal or vascular disease
other medical problems
Correct Answer: A. Multiple gestation.
C. Loss of appetite and continuous vomiting that commonly
results in dehydration and ultimately decreasing maternal 41. Question1 point(s)
nutrients.
Which of the following would the nurse assess in a client
D. Severe nausea and diarrhea that can cause experiencing abruptio placenta?
gastrointestinal irritation and possibly internal bleeding.
A. Bright red, painless vaginal bleeding
Correct Answer: B. Severe nausea and vomiting leading to
the electrolyte, metabolic, and nutritional imbalances in the B. Concealed or external dark red bleeding
absence of other medical problems. C. Palpable fetal outline
38. Question1 point(s) D. Soft and nontender abdomen
Which of the following would the nurse identify as a classic Correct Answer: B. Concealed or external dark red bleeding
sign of PIH?
42. Question1 point(s)
A. Edema of the feet and ankles
Which of the following is described as premature
B. Edema of the hands and face separation of a normally implanted placenta during the
C. Weight gain of 1 lb/week second half of pregnancy, usually with severe hemorrhage?

D. Early morning headache A. Placenta previa

Correct Answer: B. Edema of the hands and face B. Ectopic pregnancy

39. Question1 point(s) C. Incompetent cervix

In which of the following types of spontaneous abortions D. Abruptio placenta


would the nurse assess dark brown vaginal discharge and a Correct Answer: D. Abruptio placentae
negative pregnancy test?
43. Question1 point(s)
A. Threatened
Which of the following may happen if the uterus becomes
B. Imminent overstimulated by oxytocin during the induction of labor?
C. Missed A. Weak contraction prolonged to more than 70 seconds.
D. Incomplete B. Tetanic contractions prolonged to more than 90
Correct Answer: C. Missed seconds.

C. Increased pain with bright red vaginal bleeding.

D. Increased restlessness and anxiety.

Correct Answer: B. Tetanic contractions prolonged to more


than 90 seconds
Correct Answer: B. PROM removes the fetus most effective
defense against infection
44. Question1 point(s)
47. Question1 point(s)
When preparing a client for cesarean delivery, which of the
following key concepts should be considered when Which of the following factors is the underlying cause of
implementing nursing care? dystocia?

A. Instruct the mother’s support person to remain in the A. Nutritional


family lounge until after the delivery.
B. Mechanical
B. Arrange for a staff member of the anesthesia
department to explain what to expect postoperatively. C. Environmental

C. Modify preoperative teaching to meet the needs of D. Medical


either a planned or emergency cesarean birth. Incorrect
D. Explain the surgery, expected outcome, and kind of Correct Answer: B. Mechanical
anesthetics.
48. Question1 point(s)
Correct Answer: C. Modify preoperative teaching to meet
the needs of either a planned or emergency cesarean birth When uterine rupture occurs, which of the following would
be the priority?
45. Question1 point(s)
A. Limiting hypovolemic shock.
Which of the following best describes preterm labor?
B. Obtaining blood specimens.
A. Labor that begins after 20 weeks gestation and before
37 weeks gestation. C. Instituting complete bed rest.

B. Labor that begins after 15 weeks gestation and before D. Inserting a urinary catheter.
37 weeks gestation
Correct Answer: A. Limiting hypovolemic shock
C. Labor that begins after 24 weeks gestation and before
28 weeks gestation. 49. Question1 point(s)

D. Labor that begins after 28 weeks gestation and before Which of the following is the nurse’s initial action when
40 weeks gestation. umbilical cord prolapse occurs?

Correct Answer: A. Labor that begins after 20 weeks A. Begin monitoring maternal vital signs and FHR.
gestation and before 37 weeks gestation B. Place the client in a knee-chest position in bed.
46. Question1 point(s) C. Notify the physician and prepare the client for delivery.
When PROM occurs, which of the following provides D. Apply a sterile warm saline dressing to the exposed cord.
evidence of the nurse’s understanding of the client’s
immediate needs? Correct Answer: B. Place the client in a knee-chest position
in bed
A. The chorion and amnion rupture 4 hours before the
onset of labor. 50. Question1 point(s)

B. PROM removes the fetus's most effective defense Which of the following amounts of blood loss following
against infection. birth marks the criterion for describing postpartum
hemorrhage?
C. Nursing care is based on fetal viability and gestational
age. A. More than 200 ml

D. PROM is associated with malpresentation and possibly B. More than 300 ml


incompetent cervix.
C. More than 400 ml
D. More than 500 ml

Correct Answer: D. More than 500 ml 54. Question1 point(s)

Which of the following are the most commonly assessed


findings in cystitis?
51. Question1 point(s)
A. Frequency, urgency, dehydration, nausea, chills, and
Which of the following is the primary predisposing factor flank pain.
related to mastitis?
B. Nocturia, frequency, urgency, dysuria, hematuria, fever,
A. Epidemic infection from nosocomial sources localizing in and suprapubic pain.
the lactiferous glands and ducts.
C. Dehydration, Hypertension, dysuria, suprapubic pain,
B. Endemic infection occurring randomly and localizing in chills, and fever.
the peri glandular connective tissue.
D. High fever, chills, flank pain nausea, vomiting, dysuria,
C. Temporary urinary retention due to decreased and frequency.
perception of the urge to avoid.
Correct Answer: B. Nocturia, frequency, urgency dysuria,
D. Breast injury caused by overdistention, stasis, and hematuria, fever and suprapubic pain
cracking of the nipples.
55. Question1 point(s)
Correct Answer: D. Breast injury caused by overdistention,
stasis, and cracking of the nipples Which of the following best reflects the frequency of
reported postpartum “blues”?
52. Question1 point(s)
A. Between 10% and 40% of all new mothers report some
Which of the following best describes thrombophlebitis? form of postpartum blues.
A. Inflammation and clot formation that result when blood B. Between 30% and 50% of all new mothers report some
components combine to form an aggregate body. form of postpartum blues.
B. Inflammation and blood clots that eventually become C. Between 50% and 80% of all new mothers report some
lodged within the pulmonary blood vessels. form of postpartum blues.
C. Inflammation and blood clots that eventually become D. Between 25% and 70% of all new mothers report some
lodged within the femoral vein. form of postpartum blues.
D. Inflammation of the vascular endothelium with clot Correct Answer: C. Between 50% and 80% of all new
formation on the vessel wall. mothers report some form of postpartum blues
Correct Answer: D. Inflammation of the vascular 56. Question1 point(s)
endothelium with clot formation on the vessel wall
For the client who is using oral contraceptives, the nurse
53. Question1 point(s) informs the client about the need to take the pill at the
Which of the following assessment findings would the nurse same time each day to accomplish which of the following?
expect if the client develops DVT? A. Decrease the incidence of nausea.
A. Mid Calf pain, tenderness, and redness along the vein. B. Maintain hormonal levels.
B. Chills, fever, malaise, occurring 2 weeks after delivery. C. Reduce side effects.
C. Muscle pain, the presence of Homans sign, and swelling D. Prevent drug interactions.
in the affected limb.
Correct Answer: B. Maintain hormonal levels
D. Chills, fever, stiffness, and pain occurring 10 to 14 days
after delivery.

Correct Answer: C. Muscle pain the presence of Homans


sign, and swelling in the affected limb
57. Question1 point(s)

When teaching a client about contraception. Which of the 61. Question1 point(s)
following would the nurse include as the most effective
method for preventing sexually transmitted infections? Which of the following would the nurse use as the basis for
the teaching plan when caring for a pregnant teenager
A. Spermicides concerned about gaining too much weight during
pregnancy?
B. Diaphragm
A. 10 pounds per trimester.
C. Condoms
B. 1 pound per week for 40 weeks.
D. Vasectomy
C. ½ pound per week for 40 weeks.
Correct Answer: C. Condoms
D. A total gain of 25 to 30 pounds.
58. Question1 point(s)
Correct Answer: D. A total gain of 25 to 30 pounds
When preparing a woman who is 2 days postpartum for
discharge, recommendations for which of the following 62. Question1 point(s)
contraceptive methods would be avoided?
The client tells the nurse that her last menstrual period
A. Diaphragm started on January 14 and ended on January 20. Using
Nagele’s rule, the nurse determines her EDD to be which of
B. Female condom the following?
C. Oral contraceptives A. September 27
D. Rhythm method B. October 21
Correct Answer: A. Diaphragm C. November 7
59. Question1 point(s) D. December 27
For which of the following clients would the nurse expect Correct Answer: B. October 21
that an intrauterine device would not be recommended?
63. Question1 point(s)
A. Woman over age 35
When taking an obstetrical history on a pregnant client who
B. Nulliparous woman states, “I had a son born at 38 weeks gestation, a daughter
C. Promiscuous young adult born at 30 weeks gestation and I lost a baby at about 8
weeks,” the nurse should record her obstetrical history as
D. Postpartum client which of the following?

Correct Answer: C. Promiscuous young adult A. G2 T2 P0 A0 L2

60. Question1 point(s) B. G3 T1 P1 A0 L2

A client in her third trimester tells the nurse, “I’m C. G3 T2 P0 A0 L2


constipated all the time!” Which of the following should the
nurse recommend? D. G4 T1 P1 A1 L2

A. Daily enemas Correct Answer: D. G4 T1 P1 A1 L2

B. Laxatives 64. Question1 point(s)

C. Increased fiber intake When preparing to listen to the fetal heart rate at 12 weeks’
gestation, the nurse would use which of the following?
D. Decreased fluid intake
A. Stethoscope placed midline at the umbilicus.
Correct Answer: C. Increased fiber intake
B. Doppler placed midline at the suprapubic region.
C. Fetoscope placed midway between the umbilicus and 68. Question1 point(s)
the xiphoid process.
Which of the following would be the priority nursing
D. External electronic fetal monitor placed at the umbilicus. diagnosis for a client with an ectopic pregnancy?

Correct Answer: B. Doppler placed midline at the A. Risk for infection


suprapubic region
B. Pain

C. Knowledge Deficit
65. Question1 point(s)
D. Anticipatory Grieving
When developing a plan of care for a client newly diagnosed
with gestational diabetes, which of the following Correct Answer: B. Pain
instructions would be the priority? 69. Question1 point(s)
A. Dietary intake Before assessing the postpartum client’s uterus for firmness
B. Medication and position in relation to the umbilicus and midline, which
of the following should the nurse do first?
C. Exercise
A. Assess the vital signs.
D. Glucose monitoring
B. Administer analgesia.
Correct Answer: A. Dietary intake
C. Ambulate her in the hall.
66. Question1 point(s)
D. Assist her to urinate.
A client at 24 weeks gestation has gained 6 pounds in 4
weeks. Which of the following would be the priority when Correct Answer: D. Assist her to urinate
assessing the client? 70. Question1 point(s)
A. Glucosuria Which of the following should the nurse do when a
B. Depression primipara who is lactating tells the nurse that she has sore
nipples?
C. Hand/face edema
A. Tell her to breastfeed more frequently.
D. Dietary intake
B. Administer a narcotic before breastfeeding.
Correct Answer: C. Hand/face edema
C. Encourage her to wear a nursing brassiere.
67. Question1 point(s)
D. Use soap and water to clean the nipples.
A client 12 weeks’ pregnant came to the emergency
department with abdominal cramping and moderate Correct Answer: A. Tell her to breastfeed more frequently
vaginal bleeding. Speculum examination reveals 2 to 3 cm 71. Question1 point(s)
cervical dilation.The nurse would document these findings
as which of the following? The nurse assesses the vital signs of a client, 4 hours’
postpartum that are as follows: BP 90/60; temperature
A. Threatened abortion 100.4ºF; pulse 100 weak, thready; R 20 per minute. Which
B. Imminent abortion f the following should the nurse do first?

C. Complete abortion A. Report the temperature to the physician.

D. Missed abortion B. Recheck the blood pressure with another cuff.

Correct Answer: B. Imminent abortion C. Assess the uterus for firmness and position.

D. Determine the amount of lochia.

Correct Answer: D. Determine the amount of lochia


75. Question1 point(s)

72. Question1 point(s) Which of the following actions would be least effective in
maintaining a neutral thermal environment for the
The nurse assesses the postpartum vaginal discharge newborn?
(lochia) on four clients. Which of the following assessments
would warrant notification of the physician? A. Place the infant under a radiant warmer after bathing.

A. A dark red discharge on a 2-day postpartum client. B. Covering the scale with a warmed blanket prior to
weighing.
B. A pink to brownish discharge on a client who is 5 days
postpartum. C. Placing the crib close to the nursery window for family
viewing.
C. Almost colorless to creamy discharge on a client 2 weeks
after delivery. D. Covering the infant’s head with a knit stockinette.

D. A bright red discharge 5 days after delivery. Correct Answer: C. Placing the crib close to the nursery
window for family viewing.
Correct Answer: D. A bright red discharge 5 days after
delivery

73. Question1 point(s)

A postpartum client has a temperature of 101.4ºF, with a


uterus that is tender when palpated, remains unusually
large, and not descending as normally expected. Which of
the following should the nurse assess next?

A. Lochia

B. Breasts

C. Incision

D. Urine

Correct Answer: A. Lochia

74. Question1 point(s)

Which of the following is the priority focus of nursing


practice with the current early postpartum discharge?

A. Promoting comfort and restoration of health.

B. Exploring the emotional status of the family.

C. Facilitating safe and effective self and newborn care.

D. Teaching about the importance of family planning.

Correct Answer: C. Facilitating safe and effective self and


newborn care
QUIZ #2 C. Notify the physician if the skin becomes bronze in color.

A postpartum patient was in labor for 30 hours and had D. Check the vital signs every 2 to 4 hours.
ruptured membranes for 24 hours. For which of the
following would the nurse be alert? Correct Answer: D. Check the vital signs every 2 to 4 hours

A. Endometritis 5. Question1 point(s)

B. Endometriosis A primigravida in active labor is about 9 days post-term. The


client desires a bilateral pudendal block anesthesia before
C. Salpingitis delivery. After the nurse explains this type of anesthesia to
the client, which of the following locations identified by the
D. Pelvic thrombophlebitis client as the area of relief would indicate to the nurse that
Correct Answer: A. Endometritis the teaching was effective?

2. Question1 point(s) A. Back

A client at 36 weeks gestation is scheduled for a routine B. Abdomen


ultrasound prior to amniocentesis. After teaching the client C. Fundus
about the purpose of the ultrasound, which of the following
client statements would indicate to the nurse in charge that D. Perineum
the client needs further instruction?
Correct Answer: D. Perineum
A. The ultrasound will help to locate the placenta.
6. Question1 point(s)
B. The ultrasound identifies blood flow through the
umbilical cord. The nurse is caring for a primigravida at about 2 months and
1-week gestation. After explaining self-care measures for
C. The test will determine where to insert the needle. common discomforts of pregnancy, the nurse determines
that the client understands the instructions when she says:
D. The ultrasound locates a pool of amniotic fluid.
A. “Nausea and vomiting can be decreased if I eat a few
Correct Answer: B. The ultrasound identifies blood flow crackers before rising.”
through the umbilical cord.
B. “If I start to leak colostrum, I should cleanse my nipples
3. Question1 point(s) with soap and water.”
While the postpartum client is receiving heparin for C. “If I have a vaginal discharge, I should wear nylon
thrombophlebitis, which of the following drugs would the underwear.”
nurse expect to administer if the client develops
complications related to heparin therapy? D. “Leg cramps can be alleviated if I put an ice pack on the
area.”
A. Calcium gluconate
Correct Answer: A. “Nausea and vomiting can be decreased
B. Protamine sulfate if I eat a few crackers before arising”
C. Methylergonovine (Methergine) 7. Question1 point(s)
D. Nitrofurantoin (Macrodantin) Forty-eight hours after delivery, the nurse in charge plans
Correct Answer: B. Protamine sulfate discharge teaching for the client about infant care. By this
time, the nurse expects that the phase of postpartum
4. Question1 point(s) psychological adaptation that the client would be in would
be termed which of the following?
When caring for a 3-day-old neonate who is receiving
phototherapy to treat jaundice, the nurse in charge would A. Taking in
expect to do which of the following?
B. Letting go
A. Turn the neonate every 6 hours
C. Taking hold
B. Encourage the mother to discontinue breastfeeding.
D. Resolution 11. Question1 point(s)

Correct Answer: C. Taking hold A primigravida client at 25 weeks gestation visits the clinic
and tells the nurse that her lower back aches when she
arrives home from work. The nurse should suggest that the
8. Question1 point(s) client perform:

A pregnant client is diagnosed with partial placenta previa. A. Tailor sitting


In explaining the diagnosis, the nurse tells the client that the B. Leg lifting
usual treatment for partial placenta previa is which of the
following? C. Shoulder circling

A. Activity limited to bed rest. D. Squatting exercises

B. Platelet infusion. Correct Answer: A. Tailor sitting

C. Immediate cesarean delivery. 12. Question1 point(s)

D. Labor induction with oxytocin. Which of the following would the nurse in charge do first
after observing a 2-cm circle of bright red bleeding on the
Correct Answer: A. Activity limited to bed rest diaper of a neonate who just had a circumcision?
9. Question1 point(s) A. Notify the neonate’s pediatrician immediately.
The nurse plans to instruct the postpartum client about B. Check the diaper and circumcision again in 30 minutes.
methods to prevent breast engorgement. Which of the
following measures would the nurse include in the teaching C. Secure the diaper tightly to apply pressure on the site.
plan?
D. Apply gentle pressure to the site with a sterile gauze pad.
A. Feeding the neonate a maximum of 5 minutes per side
on the first day. Correct Answer: D. Apply gentle pressure to the site with a
sterile gauze pad
B. Wearing a supportive brassiere with nipple shields.
13. Question1 point(s)
C. Breastfeeding the neonate at frequent intervals.
Which of the following would the nurse most likely expect
D. Decreasing fluid intake for the first 24 to 48 hours. to find when assessing a pregnant client with abruption
placenta?
Correct Answer: C. Breastfeeding the neonate at frequent
intervals A. Excessive vaginal bleeding

10. Question1 point(s) B. Rigid, board-like abdomen

When the nurse on duty accidentally bumps the bassinet, C. Tetanic uterine contractions
the neonate throws out its arms, hands open, and begins to
cry. The nurse interprets this reaction as indicative of which D. Premature rupture of membranes
of the following reflexes? Correct Answer: B. Rigid, board-like abdomen
A. Startle reflex 14. Question1 point(s)
B. Babinski reflex While the client is in active labor with twins and the cervix
C. Grasping reflex is 5 cm dilated, the nurse observes contractions occurring
at a rate of every 7 to 8 minutes in a 30-minute period.
D. Tonic neck reflex Which of the following would be the nurse’s most
appropriate action?
Correct Answer: A. Startle reflex
A. Note the fetal heart rate patterns.

B. Notify the physician immediately.


C. Administer oxygen at 6 liters by mask. 18. Question1 point(s)

D. Have the client pant-blow during the contractions. A patient in her 14th week of pregnancy has presented with
abdominal cramping and vaginal bleeding for the past 8
Correct Answer: B. Notify the physician immediately. hours. She has passed several clots. What is the primary
15. Question1 point(s) nursing diagnosis for this patient?

A client tells the nurse, “I think my baby likes to hear me A. Knowledge deficit
talk to him.” When discussing neonates and stimulation B. Fluid volume deficit
with sound, which of the following would the nurse include
as a means to elicit the best response? C. Anticipatory grieving

A. High-pitched speech with tonal variations. D. Pain

B. Low-pitched speech with a sameness of tone. Correct Answer: B. Fluid volume deficit

C. Cooing sounds rather than words. 19. Question1 point(s)

D. Repeated stimulation with loud sounds. Immediately after delivery, the nurse-midwife assesses the
neonate’s head for signs of molding. Which factors
Correct Answer: A. High-pitched speech with tonal determine the type of molding?
variations
A. Fetal body flexion or extension
16. Question1 point(s)
B. Maternal age, body frame, and weight
A 31-year-old multipara is admitted to the birthing room
after initial examination reveals her cervix to be at 8 cm, C. Maternal and paternal ethnic backgrounds
completely effaced (100 %), and at 0 station. What phase of
labor is she in? D. Maternal parity and gravidity

A. Active phase Correct Answer: A. Fetal body flexion or extension

B. Latent phase 20. Question1 point(s)

C. Expulsive phase For a patient in active labor, the nurse-midwife plans to use
an internal electronic fetal monitoring (EFM) device. What
D. Transitional phase must occur before the internal EFM can be applied?

Correct Answer: D. Transitional phase A. The membranes must rupture.

17. Question1 point(s) B. The fetus must be at 0 station.

A pregnant patient asks the nurse if she can take castor oil C. The cervix must be dilated fully.
for her constipation. How should the nurse respond?
D. The patient must receive anesthesia.
A. “Yes, it produces no adverse effect.”
Correct Answer: A. The membranes must rupture.
B. “No, it can initiate premature uterine contractions.”
21. Question1 point(s)
C. “No, it can promote sodium retention.”
A primigravida patient is admitted to the labor delivery
D. “No, it can lead to increased absorption of fat-soluble area. Assessment reveals that she is in the early part of the
vitamins.” first stage of labor. Her pain is likely to be most intense:

Correct Answer: B. “No, it can initiate premature uterine A. Around the pelvic girdle
contractions.”
B. Around the pelvic girdle and in the upper arms

C. Around the pelvic girdle and at the perineum

D. At the perineum
Correct Answer: A. Around the pelvic girdle C. Cool incubator walls

22. Question1 point(s) D. Cool room temperature

A female adult patient is taking a progestin-only oral Correct Answer: C. Cools incubator walls
contraceptive or mini pill. Progestin use may increase the
patient’s risk for: 26. Question1 point(s)

A. Endometriosis After administering bethanechol to a patient with urine


retention, the nurse in charge monitors the patient for
B. Female hypogonadism adverse effects. Which is most likely to occur?

C. Premenstrual syndrome A. Decreased peristalsis

D. Tubal or ectopic pregnancy B. Increase heart rate

Correct Answer: D. Tubal or ectopic pregnancy C. Dry mucous membranes

D. Nausea and Vomiting

23. Question1 point(s) Correct Answer: D. Nausea and Vomiting

A patient with pregnancy-induced hypertension probably 27. Question1 point(s)


exhibits which of the following symptoms?
The nurse in charge is caring for a patient who is in the first
A. Proteinuria, headaches, vaginal bleeding stage of labor. What is the shortest but most difficult part
of this stage?
B. Headaches, double vision, vaginal bleeding
A. Active phase
C. Proteinuria, headaches, double vision
B. Complete phase
D. Proteinuria, double vision, uterine contractions
C. Latent phase
Correct Answer: C. Proteinuria, headaches, double vision
D. Transitional phase
24. Question1 point(s)
Correct Answer: D. Transitional phase
Because cervical effacement and dilation are not
progressing in a patient in labor, the doctor orders I.V. 28. Question1 point(s)
administration of oxytocin (Pitocin). Why should the nurse
monitor the patient’s fluid intake and output closely during After 3 days of breastfeeding, a postpartum patient reports
oxytocin administration? nipple soreness. To relieve her discomfort, the nurse should
suggest that she:
A. Oxytocin causes water intoxication.
A. Apply warm compresses to her nipples just before
B. Oxytocin causes excessive thirst. feeding.

C. Oxytocin is toxic to the kidneys. B. Lubricate her nipples with expressed milk before
feeding.
D. Oxytocin has a diuretic effect.
C. Dry her nipples with a soft towel after feeding.
Correct Answer: A. Oxytocin causes water intoxication.
D. Apply soap directly to her nipples, and then rinse.
25. Question1 point(s)
Correct Answer: B. Lubricate her nipples with expressed
Five hours after birth, a neonate is transferred to the milk before feeding
nursery, where the nurse intervenes to prevent
hypothermia. What is a common source of radiant heat 29. Question1 point(s)
loss?
The nurse is developing a teaching plan for a patient who is
A. Low room humidity 8 weeks pregnant. The nurse should tell the patient that she
can expect to feel the fetus move at which time?
B. Cold weight scale
33. Question1 point(s)

A. Between 10 and 12 weeks’ gestation A patient is in her last trimester of pregnancy. Nurse Vickie
should instruct her to notify her primary health care
B. Between 16 and 20 weeks’ gestation. provider immediately if she notices:
C. Between 21 and 23 weeks’ gestation. A. Blurred vision
D. Between 24 and 26 weeks’ gestation. B. Hemorrhoids
Correct Answer: B. Between 16 and 20 weeks’ gestation. C. Increased vaginal mucus
30. Question1 point(s) D. Shortness of breath on exertion
Normal lochial findings in the first 24 hours post-delivery Correct Answer: A. Blurred vision
include:
34. Question1 point(s)
A. Bright red blood
The nurse in-charge is reviewing a patient’s prenatal
B. Large clots or tissue fragments history. Which finding indicates a genetic risk factor?
C. A foul odor A. The patient is 25 years old.
D. The complete absence of lochia B. The patient has a child with cystic fibrosis.
Correct Answer: A. Bright red blood C. The patient was exposed to rubella at 36 weeks’
31. Question1 point(s) gestation.

Accompanied by her husband, a patient seeks admission to D. The patient has a history of preterm labor at 32 weeks’
the labor and delivery area. The client states that she is in gestation.
labor and says she attended the hospital clinic for prenatal Correct Answer: B. The patient has a child with cystic
care. Which question should the nurse ask her first? fibrosis
A. “Do you have any chronic illness?” 35. Question1 point(s)
B. “Do you have any allergies?” An adult female patient is using the rhythm (calendar-basal
C. “What is your expected due date?” body temperature) method of family planning. In this
method, the unsafe period for sexual intercourse is
D. “Who will be with you during labor?” indicated by:

Correct Answer: C. “What is your expected due date?” A. Return preovulatory basal body temperature.

32. Question1 point(s) B. Basal body temperature increase of 0.1 degrees to 0.2
degrees on the 2nd or 3rd day of cycle.
A patient is in the second stage of labor. During this stage,
how frequently should the nurse in charge assess her C. 3 full days of elevated basal body temperature and clear,
uterine contractions? thin cervical mucus.

A. Every 5 minutes. D. Breast tenderness and mittelschmerz.

B. Every 15 minutes. Correct Answer: C. 3 full days of elevated basal body


temperature and clear, thin cervical mucus.
C. Every 30 minutes.
36. Question1 point(s)
D. Every 60 minutes.
During a nonstress test (NST), the electronic tracing displays
Correct Answer: B. Every 15 minutes a relatively flat line for fetal movement, making it difficult
to evaluate the fetal heart rate (FHR). To mark the strip, the
nurse in charge should instruct the client to push the
control button at which time?
39. Question1 point(s)

A. At the beginning of each fetal movement. Following a precipitous delivery, examination of the client’s
vagina reveals a fourth-degree laceration. Which of the
B. At the beginning of each contraction. following would be contraindicated when caring for this
C. After every three fetal movements client?

D. At the end of fetal movement. A. Applying cold to limit edema during the first 12 to 24
hours.
Correct Answer: A. At the beginning of each fetal
movement B. Instructing the client to use two or more peri pads to
cushion the area.

C. Instructing the client on the use of sitz baths if ordered.


37. Question1 point(s)
D. Instructing the client about the importance of perineal
When evaluating a client’s knowledge of symptoms to (Kegel) exercises.
report during her pregnancy, which statement would
indicate to the nurse in charge that the client understands Correct Answer: B. Instructing the client to use two or more
the information given to her? peri pads to cushion the area

A. “I’ll report increased frequency of urination.” 40. Question1 point(s)

B. “If I have blurred or double vision, I should call the clinic A client makes a routine visit to the prenatal clinic. Although
immediately.” she is 14 weeks pregnant, the size of her uterus
approximates that in an 18- to 20-week pregnancy. Dr.
C. “If I feel tired after resting, I should report it Charles diagnoses gestational trophoblastic disease and
immediately.” orders ultrasonography. The nurse expects ultrasonoraphy
to reveal:
D. “Nausea should be reported immediately.”
A. An empty gestational sac.
Correct Answer: B. “If I have blurred or double vision, I
should call the clinic immediately.” B. Grapelike clusters.

38. Question1 point(s) C. A severely malformed fetus.

When assessing a client during her first prenatal visit, the D. An extrauterine pregnancy.
nurse discovers that the client had a reduction
mammoplasty. The mother indicates she wants to breast- Correct Answer: B. Grapelike clusters.
feed. What information should the nurse give to this 41. Question1 point(s)
moTher regarding breastfeeding success?
After completing a second vaginal examination of a client in
A. “It’s contraindicated for you to breastfeed following this labor, the nurse-midwife determines that the fetus is in the
type of surgery.” right occiput anterior position and at (–1) station. Based on
B. “I support your commitment; however, you may have to these findings, the nurse-midwife knows that the fetal
supplement each feeding with formula.” presenting part is:

C. “You should check with your surgeon to determine A. 1 cm below the ischial spines.
whether breast-feeding would be possible.” B. Directly in line with the ischial spines.
D. “You should be able to breastfeed without difficulty.” C. 1 cm above the ischial spines.
Correct Answer: B. “I support your commitment; however, D. In no relationship to the ischial spines.
you may have to supplement each feeding with formula.”
Correct Answer: C. 1 cm above the ischial spines.
42. Question1 point(s) Correct Answer: C. “What changes have you made at home
to get ready for the baby?”
Which of the following would be inappropriate to assess in
a mother who’s breastfeeding? 46. Question1 point(s)

A. The attachment of the baby to the breast. A client who’s admitted to labor and delivery has the
following assessment findings: gravida 2 para 1, estimated
B. The mother’s comfort level with positioning the baby. 40 weeks gestation, contractions 2 minutes apart, lasting 45
C. Audible swallowing. seconds, vertex +4 station. Which of the following would be
the priority at this time?
D. The baby’s lips smacking.
A. Placing the client in bed to begin fetal monitoring.
Correct Answer: D. The baby’s lips smacking
B. Preparing for immediate delivery.
43. Question1 point(s)
C. Checking for ruptured membranes.
During a prenatal visit at 4 months gestation, a pregnant
client asks whether tests can be done to identify fetal D. Providing comfort measures.
abnormalities. Between 18 and 40 weeks gestation, which Correct Answer: B. Preparing for immediate delivery.
procedure is used to detect fetal anomalies?
47. Question1 point(s)
A. Amniocentesis
The nurse is caring for a client in labor. The external fetal
B. Chorionic villi sampling monitor shows a pattern of variable decelerations in fetal
C. Fetoscopy heart rate. What should the nurse do first?

D. Ultrasound A. Change the client’s position.

Correct Answer: D. Ultrasound B. Prepare for an emergency cesarean section.

44. Question1 point(s) C. Check for placenta previa.

A client, 30 weeks pregnant, is scheduled for a biophysical D. Administer oxygen.


profile (BPP) to evaluate the health of her fetus. Her BPP Correct Answer: A. Change the client’s position.
score is 8. What does this score indicate?
48. Question1 point(s)
A. The fetus should be delivered within 24 hours.
The nurse in charge is caring for a postpartum client who
B. The client should repeat the test in 24 hours. had a vaginal delivery with a midline episiotomy. Which
C. The fetus isn’t in distress at this time. nursing diagnosis takes priority for this client?

D. The client should repeat the test in 1 week. A. Risk for deficient fluid volume related to hemorrhage.

Correct Answer: C. The fetus isn’t in distress at this time. B. Risk for infection related to the type of delivery.

45. Question1 point(s) C. Pain related to the type of incision.

A client who is 36 weeks pregnant comes to the clinic for a D. Urinary retention related to periurethral edema.
prenatal checkup. To assess the client’s preparation for Correct Answer: A. Risk for deficient fluid volume related to
parenting, the nurse might ask which question? hemorrhage
A. “Are you planning to have epidural anesthesia?” 49. Question1 point(s)
B. “Have you begun prenatal classes?” Which change would the nurse identify as a progressive
C. “What changes have you made at home to get ready for physiological change in the postpartum period?
the baby?” A. Lactation
D. “Can you tell me about the meals you typically eat each B. Lochia
day?”
C. Uterine involution 53. Question1 point(s)

D. Diuresis What is the approximate time that the blastocyst spends


traveling to the uterus for implantation?
Correct Answer: A. Lactation
A. 2 days
50. Question
B. 7 days
1 point(s)
C. 10 days
A 39-year-old at 37 weeks gestation is admitted to the
hospital with complaints of vaginal bleeding following the D. 14 weeks
use of cocaine 1 hour earlier. Which complication is most
likely causing the client’s complaint of vaginal bleeding? Correct

A. Placenta previa Correct Answer: B. 7 days

B. Abruptio placentae 54. Question1 point(s)

C. Ectopic pregnancy After teaching a pregnant woman who is in labor about the
purpose of the episiotomy, which of the following purposes
D. Spontaneous abortion stated by the client would indicate to the nurse that the
teaching was effective?
Correct Answer: B. Abruptio placentae
A. Shortens the second stage of labor.
51. Question1 point(s)
B. Enlarges the pelvic inlet.
A client with type 1 diabetes mellitus who is a multigravida
visits the clinic at 27 weeks gestation. The nurse should C. Prevents perineal edema.
instruct the client that for most pregnant women with type
1 diabetes mellitus: D. Ensures quick placenta delivery.

A. Weekly fetal movement counts are made by the mother. Correct Answer: A. Shortens the second stage of labor

B. Contraction stress testing is performed weekly. 55. Question1 point(s)

C. Induction of labor begins at 34 weeks’ gestation. A primigravida client at about 35 weeks gestation in active
labor has had no prenatal care and admitted to cocaine use
D. Nonstress testing is performed weekly until 32 weeks’ during the pregnancy. Which of the following persons must
gestation. the nurse notify?

Correct Answer: D. Nonstress testing is performed weekly A. Nursing unit manager so appropriate agencies can be
until 32 weeks’ gestation notified.

52. Question1 point(s) B. Head of the hospital’s security department.

When administering magnesium sulfate to a client with C. Chaplain in case the fetus dies in utero.
preeclampsia, the nurse understands that this drug is given
to: D. Physician who will attend the delivery of the infant.

A. Prevent seizures. Correct Answer: D. Physician who will attend the delivery of
the infant.
B. Reduce blood pressure.
56. Question1 point(s)
C. Slow the process of labor.
When preparing a teaching plan for a client who is to
D. Increase diuresis. receive a rubella vaccine during the postpartum period, the
nurse in charge should include which of the following?
Correct Answer: A. Prevent seizures
A. The vaccine prevents a future fetus from developing 60. Question1 point(s)
congenital anomalies.
While assessing a primipara during the immediate
B. Pregnancy should be avoided for 3 months after the postpartum period, the nurse in charge plans to use both
immunization. hands to assess the client’s fundus to:

C. The client should avoid contact with children diagnosed A. Prevent uterine inversion.
with rubella,
B. Promote uterine involution.
D. The injection will provide immunity against the 7-day
measles. C. Hasten the puerperium period.

Correct Answer: B. Pregnancy should be avoided for 3 D. Determine the size of the fundus.
months after the immunization Correct Answer: A. Prevent uterine inversion
57. Question1 point(s) 61. Question1 point(s)
A client with eclampsia begins to experience a seizure. Which behaviors would be exhibited during the letting-go
Which of the following would the nurse in charge do first? phase of maternal role adaptation. Select all that apply.
A. Pad the side rails. A. Emergence of the family unit.
B. Place a pillow under the left buttock. B. Dependent behaviors.
C. Insert a padded tongue blade into the mouth. C. Sexual intimacy continues.
D. Maintain a patent airway. D. Defining one's individual roles.
Correct Answer: D. Maintain a patent airway E. Being talkative and excited about becoming a mother.
58. Question1 point(s) Correct Answer: Answer: A, C, & D
While caring for a multigravida client in early labor in a 62. Question1 point(s)
birthing center, which of the following foods would be best
if the client requests a snack? While making a visit to the home of a postpartum woman 1
week after birth, the nurse should recognize that the
A. Yogurt woman would characteristically:
B. Cereal with milk A. Express a strong need to review the events and her
C. Vegetable soup behavior during the process of labor and birth.

D. Peanut butter cookies B. Exhibit a reduced attention span, limiting readiness to


learn.
Correct Answer: A. Yogurt
C. Vacillate between the desire to have her own nurturing
59. Question1 point(s) needs met and the need to take charge of her own care and
that of her newborn.
The multigravida mother with a history of rapid labor who
is in active labor calls out to the nurse, “The baby is D. Have reestablished her role as a spouse or partner.
coming!” Which of the following would be the nurse’s first
action? Correct Answer: C. Vacillate between the desire to have her
own nurturing needs met and the need to take charge of
A. Inspect the perineum. her own care and that of her newborn.

B. Time the contractions.

C. Auscultate the fetal heart rate.

D. Contact the birth attendant.

Correct Answer: A. Inspect the perineum


63. Question1 point(s) A. Biophysical

Which of the following is the most common kind of B. Psychosocial


placental adherence seen in pregnant women?
C. Geographic
A. Accreta
D. Environmental
B. Placenta previa
Correct Answer: C. Geographic
C. Percreta
67. Question1 point(s)
D. Increta
A woman who is at 36 weeks of gestation is having a
Correct Answer: A. Accreta nonstress test. Which statement indicates her correct
understanding of the test?
64. Question1 point(s)
A. "I will need to have a full bladder for the test to be done
A 40-year-old woman with a high body mass index (BMI) is accurately."
10 weeks pregnant. Which diagnostic tool is appropriate to
suggest to her at this time? B. "I should have my husband drive me home after the test
because I may be nauseated."
A. Biophysical profile
C. "This test will help to determine whether the baby has
B. Amniocentesis Down syndrome or a neural tube defect."
C. Maternal serum alpha-fetoprotein (MSAFP) D. "This test observes for fetal activity and an acceleration
D. Transvaginal ultrasound of the fetal heart rate to determine the well-being of the
baby."
Correct Answer: D. Transvaginal ultrasound
Correct Answer: D. “This test observes for fetal activity and
65. Question1 point(s) an acceleration of the fetal heart rate to determine the
well-being of the baby.”
A nurse providing care for the antepartum woman should
understand that the contraction stress test (CST): 68. Question1 point(s)

A. Sometimes uses vibroacoustic stimulation. What is an appropriate indicator for performing a


contraction stress test?
B. Is an invasive test; however, contractions are stimulated.
A. Increased fetal movement and small for gestational age.
C. Is considered to have a negative result if no late
decelerations are observed with the contractions. B. Maternal diabetes mellitus and postmaturity.

D. Is more effective than nonstress test (NST) if the C. Adolescent pregnancy and poor prenatal care.
membranes have already been ruptured.
D. History of preterm labor and intrauterine growth
Correct Answer: C. Is considered to have a negative result if restriction.
no late decelerations are observed with the contractions.
Correct Answer: B. Maternal diabetes mellitus and
66. Question1 point(s) postmaturity.

In the past, factors to determine whether a woman was 69. Question1 point(s)
likely to have a high-risk pregnancy were evaluated
primarily from a medical point of view. A broader, more The nurse sees a woman for the first time when she is 30
comprehensive approach to high-risk pregnancy has been weeks pregnant. The woman has smoked throughout the
adopted. There are now four categories based on threats to pregnancy, and fundal height measurements now are
the health of the woman and the outcome of pregnancy. suggestive of growth restriction in the fetus. In addition to
Which of the options listed here is not included as a ultrasound to measure fetal size, what would be another
category? tool useful in confirming the diagnosis?
A. Doppler blood flow analysis C. In asymmetric IUGR, weight is slightly more than SGA,
whereas length and head circumference are somewhat less
B. Contraction stress test (CST) than SGA.
C. Amniocentesis D. Symmetric IUGR occurs in the later stages of pregnancy.
D. Daily fetal movement counts Correct Answer: B. Infants with asymmetric IUGR have the
Correct Answer: A. Doppler blood flow analysis potential for normal growth and development.

70. Question1 point(s) 73. Question1 point(s)

A nurse is providing instruction for an obstetrical patient to A client who delivered by cesarean section 24 hours ago is
perform a daily fetal movement count (DFMC). Which using a patient-controlled analgesia (PCA) pump for pain
instructions could be included in the plan of care? Select all control. Her oral intake has been ice chips only since
that apply. surgery. She is now complaining of nausea and bloating,
and states that because she had nothing to eat, she is too
A. The fetal alarm signal is reached when there are no fetal weak to breastfeed her infant. Which nursing diagnosis has
movements noted for 5 hours. the highest priority?

B. The patient can monitor fetal activity once daily for a 60- A. Altered nutrition, less than body requirements for
minute period and note activity. lactation.

C. Monitor fetal activity two times a day either after meals B. Alteration in comfort related to nausea and abdominal
or before bed for a period of 2 hours or until 10 fetal distention.
movements are noted.
C. Impaired bowel motility related to pain medication and
D. Count all fetal movements in a 12-hour period daily until immobility.
10 fetal movements are noted.
D. Fatigue related to cesarean delivery and physical care
Correct Answer: B, C, & D demands of infant.

71. Question1 point(s) Correct Answer: C. Impaired bowel motility related to pain
medication and immobility
A patient has undergone an amniocentesis for evaluation of
fetal well-being. Which intervention would be included in 74. Question1 point(s)
the nurse’s plan of care after the procedure? Select all that
apply. The nurse is teaching care of the newborn to a childbirth
preparation class and describes the need for administering
A. Perform ultrasound to determine fetal positioning. antibiotic ointment into the eyes of the newborn. An
expectant father asks, “What type of disease causes
B. Observe the patient for possible uterine contractions. infections in babies that can be prevented by using this
C. Administer RhoGAM to the patient if she is Rh-negative. ointment?” Which response by the nurse is accurate?

D. Perform a mini catheterization to obtain a urine A. Herpes


specimen to assess for bleeding. B. Trichomonas
Correct Answer: B & C C. Gonorrhea
72. Question1 point(s) D. Syphilis
With regard to small-for-gestational-age (SGA) infants and Correct Answer: C. Gonorrhea
intrauterine growth restriction (IUGR), nurses should be
aware that:

A. In the first trimester, diseases or abnormalities result in


asymmetric IUGR.

B. Infants with asymmetric IUGR have the potential for


normal growth and development.
75. Question1 point(s)

A new mother is having trouble breastfeeding her newborn.


The child is making frantic rooting motions and will not
grasp the nipple. Which intervention should the nurse
implement?

A. Encourage frequent use of a pacifier so that the infant


becomes accustomed to sucking.

B. Hold the infant's head firmly against the breast until he


latches onto the nipple.

C. Encourage the mother to stop feeding for a few minutes


and comfort the infant.

D. Provide a formula for the infant until he becomes calm,


and then offer the breast again.

Correct Answer: C. Encourage the mother to stop feeding


for a few minutes and comfort the infant.
Quiz 3 A. Call the assessment data to the physician’s attention

1. Question1 point(s) B. Start oxygen per nasal cannula at 2 L/min.

A newborn who has an asymmetrical Moro reflex response C. Suction the infant’s mouth and nares.
should be further assessed for which of the following?
D. Recognize this as a normal first period of reactivity.
A. Talipes equinovarus
Correct Answer: D. Recognize this as normal first period of
B. Fractured clavicle reactivity

C. Congenital hypothyroidism 5. Question1 point(s)

D. Increased intracranial pressure The nurse hears a mother telling a friend on the telephone
about umbilical cord care. Which of the following
Correct Answer: B. Fractured clavicle statements by the mother indicates effective teaching?
2. Question1 point(s) A. “Daily soap and water cleansing is best.”
During the first 4 hours after a male circumcision, assessing B. “Alcohol helps it dry and kills germs.”
for which of the following is the priority?
C. “An antibiotic ointment applied daily prevents
A. Infection infection.”
B. Hemorrhage D. “He can have a tub bath each day.”
C. Discomfort Correct Answer: B. “Alcohol helps it dry and kills germs”
D. Dehydration 6. Question1 point(s)
Correct Answer: B. Hemorrhage A newborn weighing 3000 grams and feeding every 4 hours
3. Question1 point(s) needs 120 calories/kg of body weight every 24 hours for
proper growth and development. How many ounces of 20
The mother asks the nurse. “What’s wrong with my son’s cals/oz formula should this newborn receive at each
breasts? Why are they so enlarged?” Which of the following feeding to meet nutritional needs?
would be the best response by the nurse?
A. 2 ounces
A. “The breast tissue is inflamed from the trauma
experienced with birth.” B. 3 ounces

B. “A decrease in material hormones present before birth C. 4 ounces


causes enlargement,” D. 6 ounces
C. “You should discuss this with your doctor. It could be a Correct Answer: B. 3 ounces
malignancy.”
7. Question1 point(s)
D. “The tissue has hypertrophied while the baby was in the
uterus.” The post-term neonate with meconium-stained amniotic
fluid needs care designed to especially monitor for which of
Correct Answer: B. “A decrease in material hormones the following?
present before birth causes enlargement,”
A. Respiratory problems
4. Question1 point(s)
B. Gastrointestinal problems
Immediately after birth, the nurse notes the following on a
male newborn: respirations 78; apical heart rate 160 BPM, C. Integumentary problems
nostril-flaring; mild intercostal retractions; and grunting at
the end of expiration. Which of the following should the D. Elimination problems
nurse do? Correct Answer: A. Respiratory problems
8. Question1 point(s) D. Gluteus maximus muscle

When measuring a client’s fundal height, which of the Correct Answer: C. Vastus lateralis muscle
following techniques denotes the correct method of
measurement used by the nurse? 12. Question1 point(s)

A. From the xiphoid process to the umbilicus. When performing a pelvic examination, the nurse observes
a red swollen area on the right side of the vaginal orifice.
B. From the symphysis pubis to the xiphoid process. The nurse would document this as enlargement of which of
the following?
C. From the symphysis pubis to the fundus.
A. Clitoris
D. From the fundus to the umbilicus.
B. Parotid gland
Correct Answer: C. From the symphysis pubis to the fundus.
C. Skene’s gland
9. Question1 point(s)
D. Bartholin’s gland
A client with severe preeclampsia is admitted with BP
160/110, proteinuria, and severe pitting edema. Which of Correct Answer: D. Bartholin’s gland
the following would be most important to include in the
client’s plan of care? 13. Question1 point(s)

A. Daily weights To differentiate as a female, the hormonal stimulation of


the embryo that must occur involves which of the
B. Seizure precautions following?

C. Right lateral positioning A. Increase in maternal estrogen secretion

D. Stress reduction B. Decrease in maternal androgen secretion.

Correct Answer: B. Seizure precautions C. Secretion of androgen by the fetal gonad.

10. Question1 point(s) D. Secretion of estrogen by the fetal gonad.

A postpartum primipara asks the nurse, “When can we have Correct Answer: D. Secretion of estrogen by the fetal gonad.
sexual intercourse again?” Which of the following would be
the nurse’s best response? 14. Question1 point(s)

A. “Anytime you both want to.” A client at 8 weeks’ gestation calls complaining of slight
nausea in the morning hours. Which of the following client
B. “As soon as you choose a contraceptive method.” interventions should the nurse question?

C. “When the discharge has stopped, and the incision is A. Taking 1 teaspoon of bicarbonate of soda in an 8-ounce
healed.” glass of water.

D. “After your 6 weeks examination.” B. Eating a few low-sodium crackers before getting out of
bed.
Correct Answer: C. “When the discharge has stopped and
the incision is healed.” C. Avoiding the intake of liquids in the morning hours.

11. Question1 point(s) D. Eating six small meals a day instead of three large meals.

When preparing to administer the vitamin K injection to a Correct Answer: A. Taking 1 teaspoon of bicarbonate of
neonate, the nurse would select which of the following sites soda in an 8-ounce glass of water.
as appropriate for the injection?

A. Deltoid muscle

B. Anterior femoris muscle

C. Vastus lateralis muscle


15. Question1 point(s)

The nurse documents positive ballottement in the client’s A. Obtaining an order to begin IV oxytocin infusion.
prenatal record. The nurse understands that this indicates
which of the following? B. Administering a light sedative to allow the patient to rest
for several hours
A. Palpable contractions on the abdomen.
C. Preparing for a cesarean section for failure to progress
B. Passive movement of the unengaged fetus.
Correct Answer: A. Obtaining an order to begin IV oxytocin
C. Fetal kicking felt by the client. infusion.

D. Enlargement and softening of the uterus. 19. Question1 point(s)

Correct Answer: B. Passive movement of the unengaged A multigravida at 38 weeks’ gestation is admitted with
fetus. painless, bright red bleeding and mild contractions every 7
to 10 minutes. Which of the following assessments should
16. Question1 point(s) be avoided?
During a pelvic exam, the nurse notes a purple-blue tinge of A. Maternal vital sign
the cervix. The nurse documents this as which of the
following? B. Fetal heart rate

A. Braxton-Hicks sign C. Contraction monitoring

B. Chadwick’s sign D. Cervical dilation

C. Goodell’s sign Correct Answer: D. Cervical dilation

D. McDonald’s sign 20. Question1 point(s)

Correct Answer: B. Chadwick’s sign Which of the following would be the nurse’s most
appropriate response to a client who asks why she must
17. Question1 point(s) have a cesarean delivery if she has a complete placenta
During a prenatal class, the nurse explains the rationale for previa?
breathing techniques during preparation for labor based on A. “You will have to ask your physician when he returns.”
the understanding that breathing techniques are most
important in achieving which of the following? B. “You need a cesarean to prevent hemorrhage.”

A. Eliminate pain and give expectant parents something to C. “The placenta is covering most of your cervix.”
do.
D. “The placenta is covering the opening of the uterus and
B. Reduce the risk of fetal distress by increasing blocking your baby.”
uteroplacental perfusion.
Correct Answer: D. “The placenta is covering the opening of
C. Facilitate relaxation, possibly reducing the perception of the uterus and blocking your baby.”
pain.
21. Question1 point(s)
D. Eliminate pain so that less analgesia and anesthesia are
needed. The nurse understands that the fetal head is in which of the
following positions with a face presentation?
Correct Answer: C. Facilitate relaxation, possibly reducing
the perception of pain. A. Completely flexed

18. Question1 point(s) B. Completely extended

After 4 hours of active labor, the nurse notes that the C. Partially extended
contractions of a primigravida client are not strong enough D. Partially flexed
to dilate the cervix. Which of the following would the nurse
anticipate doing? Correct Answer: B. Completely extended
22. Question1 point(s) D. Sharing of a common chorion.

With a fetus in the left anterior breech presentation, the Correct Answer: A. Two ova fertilized by separate sperm
nurse would expect the fetal heart rate would be most
audible in which of the following areas? 26. Question1 point(s)

Which of the following refers to the single cell that


reproduces itself after conception?
A. Above the maternal umbilicus and to the right of
midline. A. Chromosome

B. In the lower-left maternal abdominal quadrant. B. Blastocyst

C. In the lower-right maternal abdominal quadrant. C. Zygote

D. Above the maternal umbilicus and to the left of the D. Trophoblast


midline. Correct Answer: C. Zygote
Correct Answer: D. Above the maternal umbilicus and to the 27. Question1 point(s)
left of midline
In the late 1950s, consumers and health care professionals
23. Question1 point(s) began challenging the routine use of analgesics and
The amniotic fluid of a client has a greenish tint. The nurse anesthetics during childbirth. Which of the following was an
interprets this to be the result of which of the following? outgrowth of this concept?

A. Lanugo A. Labor, delivery, recovery, postpartum (LDRP)

B. Hydramnios B. Nurse-midwifery

C. Meconium C. Clinical nurse specialist

D. Vernix D. Prepared childbirth

Correct Answer: C. Meconium Correct Answer: D. Prepared childbirth

24. Question1 point(s) 28. Question1 point(s)

A patient is in labor and has just been told she has a breech A client has a mid pelvic contracture from a previous pelvic
presentation. The nurse should be particularly alert for injury due to a motor vehicle accident as a teenager. The
which of the following? nurse is aware that this could prevent a fetus from passing
through or around which structure during childbirth?
A. Quickening
A. Symphysis pubis
B. Ophthalmia neonatorum
B. Sacral promontory
C. Pica
C. Ischial spines
D. Prolapsed umbilical cord
D. Pubic arch
Correct Answer: D. Prolapsed umbilical cord
Correct Answer: C. Ischial spines
25. Question1 point(s)
29. Question1 point(s)
When describing dizygotic twins to a couple, on which of
the following would the nurse base the explanation? When teaching a group of adolescents about variations in
the length of the menstrual cycle, the nurse understands
A. Two ova fertilized by separate sperm. that the underlying mechanism is due to variations in which
of the following phases?
B. Sharing of a common placenta.
A. Menstrual phase
C. Each ova with the same genotype.
B. Proliferative phase
C. Secretory phase 33. Question1 point(s)

D. Ischemic phase The infant of a substance-abusing mother is at risk for


developing a sense of which of the following?
Correct Answer: B. Proliferative phase
A. Mistrust
30. Question1 point(s)
B. Shame
When teaching a group of adolescents about male hormone
production, which of the following would the nurse include C. Guilt
as being produced by the Leydig cells?
D. Inferiority
A. Follicle-stimulating hormone
Correct Answer: A. Mistrust
B. Testosterone

C. Luteinizing hormone
34. Question1 point(s)
D. Gonadotropin-releasing hormone
Which of the following toys should the nurse recommend
Correct Answer: B. Testosterone for a 5-month-old?

31. Question1 point(s) A. A big red balloon

While performing a physical assessment of a 12 month-old, B. A teddy bear with button eyes
the nurse notes that the infant’s anterior fontanel is still
slightly open. Which of the following is the nurse’s most C. A push-pull wooden truck
appropriate action? D. A colorful busy box
A. Notify the physician immediately because there is a Correct Answer: D. A colorful busy box
problem.
35. Question1 point(s)
B. Perform an intensive neurological examination.
The mother of a 2-month-old is concerned that she may be
C. Perform an intensive developmental examination. spoiling her baby by picking her up when she cries. Which
D. Do nothing because this is a normal finding for the age. of the following would be the nurse’s best response?

Correct Answer: D. Do nothing because this is a normal A. “ Let her cry for a while before picking her up, so you
finding for the age. don’t spoil her.”

32. Question1 point(s) B. “Babies need to be held and cuddled; you won’t spoil her
this way.”
When teaching a mother about introducing solid foods to
her child, which of the following indicates the earliest age C. “Crying at this age means the baby is hungry; give her a
at which this should be done? bottle.”

A. 1 month D. “If you leave her alone she will learn how to cry herself
to sleep.”
B. 2 months
Correct Answer: B. “Babies need to be held and cuddled;
C. 3 months you won’t spoil her this way.”

D. 4 months 36. Question1 point(s)

Correct Answer: D. 4 months When assessing an 18-month-old, the nurse notes a


characteristic protruding abdomen. Which of the following
would explain the rationale for this finding?

A. Increased food intake owing to age.

B. Underdeveloped abdominal muscles.


C. Bow Legged posture. D. Increase in appetite

D. Linear growth curve. Correct Answer: A. Food “jags.”

Correct Answer: B. Underdeveloped abdominal muscles 41. Question1 point(s)

37. Question1 point(s) Which of the following suggestions should the nurse offer
the parents of a 4-year-old boy who resists going to bed at
If parents keep a toddler dependent in areas where he is night?
capable of using skills, the toddler will develop a sense of
which of the following? A. “Allow him to fall asleep in your room, then move him to
his own bed.”
A. Mistrust
B. “Tell him that you will lock him in his room if he gets out
B. Shame of bed one more time.”
C. Guilt C. “Encourage active play at bedtime to tire him out so he
D. Inferiority will fall asleep faster.”

Correct Answer: B. Shame D. “Read him a story and allow him to play quietly in his
bed until he falls asleep.”
38. Question1 point(s)
Correct Answer: D. “Read him a story and allow him to play
Which of the following is an appropriate toy for an 18- quietly in his bed until he falls asleep.”
month-old?
42. Question1 point(s)
A. Multiple-piece puzzle
When providing therapeutic play, which of the following
B. Miniature cars toys would best promote imaginative play in a 4-year-old?

C. Finger paints A. Large blocks

D. Comic book B. Dress-up clothes

Correct Answer: C. Finger paints C. Wooden puzzle

39. Question1 point(s) D. Big wheels

When teaching parents about the child’s readiness for toilet Correct Answer: B. Dress-up clothes
training, which of the following signs should the nurse
instruct them to watch for in the toddler? 43. Question1 point(s)

A. Demonstrates dryness for 4 hours. Which of the following activities, when voiced by the
parents following a teaching session about the
B. Demonstrates ability to sit and walk. characteristics of school-age cognitive development would
indicate the need for additional teaching?
C. Has a new sibling for stimulation.
A. Collecting baseball cards and marbles.
D. Verbalizes desire to go to the bathroom.
B. Ordering dolls according to size.
Correct Answer: D. Verbalizes desire to go to the bathroom
C. Considering simple problem-solving options.
40. Question1 point(s)
D. Developing plans for the future.
When teaching parents about typical toddler eating
patterns, which of the following should be included? Correct Answer: D. Developing plans for the future

A . Food “jags.”

B. Preference to eat alone

C. Consistent table manners


44. Question1 point(s) B. In a year from now

A hospitalized school age child states: “I’m not afraid of this C. At age 10
place, I’m not afraid of anything.” This statement is most
likely an example of which of the following? D. At age 13

A. Regression Correct Answer: C. At age 10

B. Repression 48. Question1 point(s)

C. Reaction formation The adolescent’s inability to develop a sense of who he is


and what he can become results in the sense of which of
D. Rationalization the following?

Correct Answer: C. Reaction formation A. Shame

45. Question1 point(s) B. Guilt

After teaching a group of parents about accident C. Inferiority


prevention for school agers, which of the following
statements by the group would indicate the need for more D. Role confusion
teaching? Correct Answer: D. Role confusion
A. “School Agers are more active and adventurous than According to Erikson, role cunfusion develops when the
younger children.” adolescent does not develop a sense of identity and a sense
B. “School Agers are more susceptible to home hazards of where he fits in.
than are younger children.” 49. Question1 point(s)
C. “Schoolagers are unable to understand potential Which of the following would be most appropriate for a
dangers around them.” nurse to use when describing menarche to a 13-year-old?
D. “Schoolargers are less subject to parental control than A. A female’s first menstruation or menstrual “periods.”
are younger children.”
B. The first year of menstruation or “period."
Correct Answer: C. “Schoolagers are unable to understand
potential dangers around them.” C. The entire menstrual cycle or from one “period” to
another.
46. Question1 point(s)
D. The onset of uterine maturation or peak growth.
Which of the following skills is the most significant one
learned during the school-age period? Correct Answer: A. A female’s first menstruation or
menstrual “periods”.
A. Collecting
50. Question1 point(s)
B. Ordering
A 14-year-old boy has acne and according to his parents,
C. Reading dominates the bathroom by using the mirror all the time.
D. Sorting Which of the following remarks by the nurse would be least
helpful in talking to the boy and his parents?
Correct Answer: C. Reading
A. “This is probably the only concern he has about his body.
47. Question1 point(s) So don’t worry about it or the time he spends on it.”

A child age 7 was unable to receive the measles, mumps, B. “Teenagers are anxious about how their peers perceive
and rubella (MMR) vaccine at the recommended scheduled them. So they spend a lot of time grooming.”
time. When would the nurse expect to administer the MMR
vaccine? C. “A teen may develop a poor self-image when
experiencing acne. Do you feel this way sometimes?”
A. In a month from now
D. “You appear to be keeping your face well washed. When assessing a child with a cleft palate, the nurse is
Would you feel comfortable discussing your cleansing aware that the child is at risk for more frequent episodes of
method?” otitis media due to which of the following?

Correct Answer: A. “This is probably the only concern he has A. Lowered resistance from malnutrition.
about his body. So don’t worry about it or the time he
spends on it.” B. Ineffective functioning of the Eustachian tubes.

51. Question1 point(s) C. Plugging of the Eustachian tubes with food particles.

Which of the following should the nurse suspect when D. Associated congenital defects of the middle ear.
noting that a 3-year-old is engaging in explicit sexual Correct Answer: B. Ineffective functioning of the Eustachian
behavior during doll play? tubes
A. The child is exhibiting normal preschool curiosity. 55. Question1 point(s)
B. The child is acting out personal experiences. While performing a neurodevelopmental assessment on a
C. The child does not know how to play with dolls. 3-month-old infant, which of the following characteristics
would be expected?
D. The child is probably developmentally delayed.
A. A strong Moro reflex.
Correct Answer: B. The child is acting out personal
experiences B. A strong parachute reflex.

52. Question1 point(s) C. Rolling from front to back.

Which of the following statements by the parents of a child D. Lifting of head and chest when prone.
with school phobia would indicate the need for further Correct Answer: D. Lifting of head and chest when prone
teaching?
56. Question1 point(s)
A. “We’ll keep him at home until the phobia subsides.”
By the end of which of the following would the nurse most
B. “We’ll work with his teachers and counselors at school.” commonly expect a child’s birth weight to triple?
C. “We’ll try to encourage him to talk about his problem.” A. 4 months
D. “We’ll discuss possible solutions with him and his B. 7 months
counselor.”
C. 9 months
Correct Answer: A. “We’ll keep him at home until the
phobia subsides.” D. 12 months

53. Question1 point(s) Correct Answer: D. 12 months

When developing a teaching plan for a group of high school 57. Question1 point(s)
students about teenage pregnancy, the nurse would keep
in mind which of the following? Which of the following best describes parallel play between
two toddlers?
A. The incidence of teenage pregnancies is increasing.
A. Sharing crayons to color separate pictures.
B. Most teenage pregnancies are planned.
B. Playing a board game with a nurse.
C. Denial of the pregnancy is common early on.
C. Sitting near each other while playing with separate dolls.
D. The risk for complications during pregnancy is rare.
D. Sharing their dolls with two different nurses.
Correct Answer: C. Denial of the pregnancy is common early
on. Correct Answer: C. Sitting near each other while playing
with separate dolls
54. Question1 point(s)
58. Question1 point(s)
Which of the following would the nurse identify as the initial Correct Answer: B. Projection
priority for a child with acute lymphocytic leukemia?
62. Question1 point(s)
A. Instituting infection control precautions.
Which of the following should the nurse expect to note as a
B. Encouraging adequate intake of iron-rich foods. frequent complication for a child with congenital heart
disease?
C. Assisting with coping with chronic illness.
A. Susceptibility to respiratory infection
D. Administering medications via IM injections.
B. Bleeding tendencies
Correct Answer: A. Instituting infection control precautions
C. Frequent vomiting and diarrhea
59. Question1 point(s)
D. Seizure disorder
Which of the following information, when voiced by the
mother, would indicate to the nurse that she understands Correct Answer: A. Susceptibility to respiratory infection
home care instructions following the administration of
diphtheria, tetanus, and pertussis injection? 63. Question1 point(s)

A. Measures to reduce fever. Which of the following would the nurse do first for a 3-year-
old boy who arrives in the emergency room with a
B. Need for dietary restrictions. temperature of 105 degrees, inspiratory stridor, and
restlessness, who is leaning forward and drooling?
C. Reasons for subsequent rash.
A. Auscultate his lungs and place him in a mist tent.
D. Measures to control subsequent diarrhea.
B. Have him lie down and rest after encouraging fluids.
Correct Answer: A. Measures to reduce fever
C. Examine his throat and perform a throat culture.
60. Question1 point(s)
D. Notify the physician immediately and prepare for
Which of the following actions by a community health nurse intubation.
is most appropriate when noting multiple bruises and burns
on the posterior trunk of an 18-month-old child during a Correct Answer: D. Notify the physician immediately and
home visit? prepare for intubation.

A. Report the child’s condition to Protective Services 64. Question1 point(s)


immediately.
Which of the following would the nurse need to keep in
B. Schedule a follow-up visit to check for more bruises. mind as a predisposing factor when formulating a teaching
plan for a child with a urinary tract infection?
C. Notify the child’s physician immediately.
A. A shorter urethra in females.
D. Don nothing because this is a normal finding in a toddler.
B. Frequent emptying of the bladder.
Correct Answer: A. Report the child’s condition to
Protective Services immediately. C. Increased fluid intake.

61. Question1 point(s) D. Ingestion of acidic juices.

Which of the following is being used when the mother of a Correct Answer: A. A shorter urethra in females
hospitalized child calls the student nurse and states, “You
idiot, you have no idea how to care for my sick child”? 65. Question1 point(s)

A. Displacement Which of the following should the nurse do first for a 15-
year-old boy with a full leg cast who is screaming in
B. Projection unrelenting pain and exhibiting right foot pallor signifying
compartment syndrome?
C. Repression
A. Medicate him with acetaminophen.
D. Psychosis
B. Notify the physician immediately. A. Slow to feed self.

C. Release the traction. B. Lack of speech.

D. Monitor him every 5 minutes. C. Marked motor delays.

Correct Answer: B. Notify the physician immediately D. Gait disability.

66. Question1 point(s) Correct Answer: A. Slow to feed self.

At which of the following ages would the nurse expect to 70. Question1 point(s)
administer the varicella zoster vaccine to a child?
Which of the following assessment findings would lead the
A. At birth nurse to suspect Down syndrome in an infant?

B. 2 months A. Small tongue

C. 6 months B. Transverse palmar crease

D. 12 months C. Large nose

Correct Answer: D. 12 months D. Restricted joint movement

67. Question1 point(s) Correct Answer: B. Transverse palmar crease

When discussing normal infant growth and development 71. Question1 point(s)
with parents, which of the following toys would the nurse
suggest as most appropriate for an 8-month-old? While assessing a newborn with cleft lip, the nurse would
be alert that which of the following will most likely be
A. Push-pull toys compromised?

B. Rattle A. Sucking ability

C. Large blocks B. Respiratory status

D. Mobile C. Locomotion

Correct Answer: C. Large blocks D. GI function

68. Question1 point(s) Correct Answer: A. Sucking ability

Which of the following aspects of psychosocial 72. Question1 point(s)


development is necessary for the nurse to keep in mind
when providing care for the preschool child? When providing postoperative care for the child with a cleft
palate, the nurse should position the child in which of the
A. The child can use complex reasoning to think out of following positions?
situations.
A. Supine
B. Fear of body mutilation is a common preschool fear.
B. Prone
C. The child engages in competitive types of play.
C. In an infant seat
D. Immediate gratification is necessary to develop
initiative. D. On the side

Correct Answer: B. Fear of body mutilation is a common Correct Answer: B. Prone


preschool fear

69. Question1 point(s) 73. Question1 point(s)


Which of the following is characteristic of a preschooler While assessing a child with pyloric stenosis, the nurse is
with mild-mental retardation? likely to note which of the following?
A. Regurgitation

B. Steatorrhea

C. Projectile vomiting

D. “Currant jelly” stools

Correct Answer: C. Projectile vomiting

74. Question1 point(s)

Which of the following nursing diagnoses would be


inappropriate for the infant with gastroesophageal reflux
(GER)?

A. Fluid volume deficit

B. Risk for aspiration

C. Altered nutrition: less than body requirements

D. Altered oral mucous membranes

Correct Answer: D. Altered oral mucous membranes

75. Question1 point(s)

Which of the following parameters would the nurse


monitor to evaluate the effectiveness of thickened feedings
for an infant with gastroesophageal reflux disease (GERD)?

A. Vomiting

B. Stools

C. Uterine

D. Weight

Correct

Correct Answer: A. Vomiting

\
Correct Answer: A. Hirschsprung disease

1. Question1 point(s) 5. Question1 point(s)

Discharge teaching for a child with celiac disease would When assessing a child for possible intussusception, which
include instructions about avoiding which of the following? of the following would be least likely to provide valuable
information?
A. Rice
A. Stool inspection
B. Milk
B. Pain pattern
C. Wheat
C. Family history
D. Chicken
D. Abdominal palpation
Correct Answer: C. Wheat
Correct Answer: C. Family history
2. Question1 point(s)
6. Question1 point(s)
Which of the following would the nurse expect to assess in
a child with celiac disease having a celiac crisis secondary to Nurse Barry is performing Leopold’s maneuver and found
an upper respiratory infection? the following: breech presentation, fetal back at the right
side of the mother. Based on these findings, the nurse can
A. Respiratory distress hear the fetal heartbeat (PMI) best in which location?
B. Lethargy A. Left lower quadrant
C. Watery diarrhea B. Right lower quadrant
D. Weight gain C. Left upper quadrant
Correct Answer: C. Watery diarrhea D. Right upper quadrant
3. Question1 point(s) Correct Answer: B. Right lower quadrant
Which of the following should the nurse do first after noting 7. Question1 point(s)
that a child with Hirschsprung disease has a fever and
watery explosive diarrhea? In Leopold’s maneuver step #1, the nurse palpated a soft,
broad mass that moves with the rest of the mass. The
A. Notify the physician immediately. correct interpretation of this finding is:
B. Administer antidiarrheal medications. A. The mass palpated at the fundal part is the head part.
C. Monitor child every 30 minutes. B. The presentation is breech.
D. Nothing, this is characteristic of Hirschsprung disease. C. The mass palpated is the back.
Correct Answer: A. Notify the physician immediately. D. The mass palpated is the buttocks.
4. Question1 point(s) Correct Answer: D. The mass palpated is the buttocks.
A newborn’s failure to pass meconium within the first 24 8. Question1 point(s)
hours after birth may indicate which of the following?
In Leopold’s maneuver step #3, the nurse palpated a hard
A. Hirschsprung disease round movable mass at the suprapubic area. The correct
B. Celiac disease interpretation is that the mass palpated is:

C. Intussusception A. The buttocks because the presentation is breech.

D. Abdominal wall defect B. The mass palpated is the head.


C. The mass is the fetal back. D. The woman may be experiencing complications of
pregnancy.
D. The mass palpated is the small fetal part.
Correct Answer: B. The fundus of the uterus is high pushing
Correct Answer: B. The mass palpated is the head. the diaphragm upwards
9. Question1 point(s) 13. Question1 point(s)
The hormone responsible for a positive pregnancy test is: Which of the following findings in a woman would be
A. Estrogen consistent with a pregnancy of two months duration?

B. Progesterone A. Weight gain of 6-10 lbs. And the presence of striae


gravidarum.
C. Human Chorionic Gonadotropin
B. Fullness of the breast and urinary frequency.
D. Follicle Stimulating Hormone
C. Braxton Hicks contractions and quickening.
Correct Answer: C. Human Chorionic Gonadotropin
D. Increased respiratory rate and ballottement.
10. Question1 point(s)
Correct Answer: B. Fullness of the breast and urinary
The hormone responsible for the maturation of the frequency.
Graafian follicle is
14. Question1 point(s)
A. Follicle-stimulating hormone
Which of the following is a positive sign of pregnancy?
B. Progesterone
A. Fetal movement felt by mother
C. Estrogen
B. Enlargement of the uterus
D. Luteinizing hormone
C. (+) pregnancy test
Correct Answer: A. Follicle-stimulating hormone
D. (+) ultrasound
11. Question1 point(s)
Correct Answer: D. (+) ultrasound
The most common normal position of the fetus in utero is:
15. Question1 point(s)
A. Transverse position
What event occurring in the second trimester helps the
B. Vertical position expectant mother to accept the pregnancy?

C. Oblique position A. Lightening

D. None of the above B. Ballotment

Correct Answer: B. Vertical position C. Pseudocyesis

12. Question1 point(s) D. Quickening

In the later part of the 3rd trimester, the mother may Correct Answer: D. Quickening
experience shortness of breath. This complaint may be
explained as: 16. Question1 point(s)

A. A normal occurrence in pregnancy because the fetus is Shoes with low, broad heels, plus a good posture will
using more oxygen. prevent which prenatal discomfort?

B. The fundus of the uterus is high pushing the diaphragm A. Backache


upwards. B. Vertigo
C. The woman is having an allergic reaction to the C. Leg cramps
pregnancy and its hormones.
D. Nausea C. 7th month

Correct Answer: A. Backache D. 8th month

17. Question1 point(s) Correct Answer: C. 7th month

When a pregnant woman experiences leg cramps, the 21. Question1 point(s)
correct nursing intervention to relieve the muscle cramps
is: The following are ways of determining the expected date of
delivery (EDD) when the LMP is unknown EXCEPT:
A. Allow the woman to exercise.
A. Naegele’s rule
B. Let the woman walk for a while.
B. Quickening
C. Let the woman lie down and dorsiflex the foot towards
the knees. C. McDonald’s rule

D. Ask the woman to raise her legs. D. Batholomew’s rule of 4

Correct Answer: C. Let the woman lie down and dorsiflex Correct Answer: A. Naegele’s rule
the foot towards the knees 22. Question1 point(s)
18. Question1 point(s) If the LMP is Jan. 30, the expected date of delivery (EDD) is:
From the 33rd week of gestation till full term, a healthy A. Oct. 7
mother should have a prenatal check-up every:
B. Oct. 24
A. Week
C. Nov. 7
B. 2 weeks
D. Nov. 8
C. 3 weeks
Correct Answer: C. Nov. 7
D. 4 weeks
23. Question1 point(s)
Correct Answer: A. Week
Kegel’s exercise is done in pregnancy in order to:
19. Question1 point(s)
A. Strengthen perineal muscles.
The expected weight gain in a normal pregnancy during the
3rd trimester is: B. Relieve backache.

A. 1 pound a week C. Strengthen abdominal muscles.

B. 2 pounds a week D. Prevent leg varicosities and edema.

C. 10 lbs a month Correct Answer: A. Strengthen perineal muscles

D. 10 lbs total weight gain in the 3rd trimester 24. Question1 point(s)

Incorrect Pelvic rocking is an appropriate exercise in pregnancy to


relieve which discomfort?
Correct Answer: A. 1 pound a week
A. Leg cramps
20. Question1 point(s)
B. Urinary frequency
In Bartholomew’s Rule of 4, when the level of the fundus is
midway between the umbilicus and xiphoid process the C. Orthostatic hypotension
estimated age of gestation (AOG) is:
D. Backache
A. 5th month
Correct Answer: D. Backache
B. 6th month
A. Asking her to void.

B. Taking her vital signs and recording the readings.

25. Question1 point(s) C. Giving the client perineal care.

The main reason for an expected increased need for iron in D. Doing a vaginal prep.
pregnancy is:
Correct Answer: A. Asking her to void.
A. The mother may have physiologic anemia due to the
increased need for red blood cell mass as well as the fetal 29. Question1 point(s)
requires about 350-400 mg of iron to grow. When preparing the mother who is in her 4th month of
B. The mother may suffer anemia because of poor appetite. pregnancy for an abdominal ultrasound, the nurse should
instruct her to:
C. The fetus has an increased need for RBC which the
mother must supply. A. Observe NPO from midnight to avoid vomiting.

D. The mother may have a problem with digestion because B. Do perineal flushing properly before the procedure.
of pica. C. Drink at least 2 liters of fluid 2 hours before the
Correct Answer: A. The mother may have physiologic procedure and not void until the procedure is done.
anemia due to the increased need for red blood cell mass, D. Void immediately before the procedure for better
as well as the fetal, requires about 350-400 mg of iron to visualization.
grow.
Correct Answer: C. Drink at least 2 liters of fluid 2 hours
26. Question1 point(s) before the procedure and not void until the procedure is
The diet that is appropriate in normal pregnancy should be done.
high in: 30. Question1 point(s)
A. Protein, minerals, and vitamins The nursing intervention to relieve “morning sickness” in a
B. Carbohydrates and vitamins pregnant woman is by giving:

C. Proteins, carbohydrates, and fats A. Dry carbohydrate food like crackers

D. Fats and minerals B. Low sodium diet

Correct Answer: A. Protein, minerals, and vitamins C. Intravenous infusion

27. Question1 point(s) D. Antacid

Which of the following signs will require a mother to seek Correct Answer: A. Dry carbohydrate food like crackers
immediate medical attention? 31. Question1 point(s)
A. When the first fetal movement is felt. The common normal site of nidation/implantation in the
B. No fetal movement is felt on the 6th month. uterus is:

C. Mild uterine contraction. A. Upper uterine portion

D. Slight dyspnea on the last month of gestation. B. Mid-uterine area

Correct Answer: B. No fetal movement is felt on the 6th C. Lower uterine segment
month. D. Lower cervical segment
28. Question1 point(s) Correct Answer: A. Upper uterine portion
You want to perform a pelvic examination on one of your 32. Question1 point(s)
pregnant clients. You prepare your client for the procedure
by:
Mrs. Santos is on her 5th pregnancy and has a history of 36. Question1 point(s)
abortion in the 4th pregnancy, and the first pregnancy was
a twin. She is considered to be: You are performing an abdominal exam on a 9th-month
pregnant woman. While lying supine, she felt breathless,
had pallor, tachycardia, and cold clammy skin. The correct
assessment of the woman’s condition is that she is:
A. G 4 P 3

B. G 5 P 3
A. Experiencing the beginning of labor.
C. G 5 P 4
B. Having supine hypotension.
D. G 4 P 4
C. Having sudden elevation of BP.
Correct Answer: B. G 5 P 3
D. Going into shock.
33. Question1 point(s)
Correct Answer: B. Having supine hypotension.
The following are skin changes in pregnancy EXCEPT:
37. Question1 point(s)
A. Chloasma
Smoking is contraindicated in pregnancy because:
B. Striae gravidarum
A. Nicotine causes vasodilation of the mother’s blood
C. Linea negra vessels.
D. Chadwick’s sign B. Carbon monoxide binds with the hemoglobin of the
Correct Answer: D. Chadwick’s sign mother reducing available hemoglobin for the fetus.

34. Question1 point(s) C. The smoke will make the fetus, and the mother feels
dizzy.
Which of the following statements is TRUE of conception?
D. Nicotine will cause vasoconstriction of the fetal blood
A. Within 2-4 hours after intercourse, conception is vessels.
possible in a fertile woman.
Correct Answer: B. Carbon monoxide binds with the
B. Generally, fertilization is possible 4 days after ovulation. hemoglobin of the mother reducing available hemoglobin
for the fetus.
C. Conception is possible during menstruation in a long
menstrual cycle. 38. Question1 point(s)

D. To avoid conception, intercourse must be avoided 5 days Which of the following is the most likely effect on the fetus
before and 3 days after menstruation. if the woman is severely anemic during pregnancy?

Correct Answer: A. Within 2-4 hours after intercourse A. Large for gestational age (LGA) fetus
conception is possible in a fertile woman.
B. Hemorrhage
35. Question1 point(s)
C. Small for gestational age (SGA) baby
Which of the following are the functions of amniotic fluid?
Select all that apply. D. Erythroblastosis fetalis

A. Cushions the fetus from abdominal trauma 39. Question1 point(s)

B. Serves as the fluid for the fetus Which of the following signs and symptoms will most likely
make the nurse suspect that the patient has hydatidiform
C. Maintains the internal temperature mole?

D. Facilitates fetal movement A. Slight bleeding

Correct Answer: A, B, C, & D B. Passage of clear vesicular mass per vagina


C. Absence of fetal heartbeat Correct Answer: B. Minimize oxygen consumption which
can aggravate the condition of the compromised heart of
D. Enlargement of the uterus the mother.
Correct Answer: B. Passage of clear vesicular mass per 43. Question1 point(s)
vagina
A pregnant mother is admitted to the hospital with the chief
complaint of profuse vaginal bleeding, AOG 36 wks, not in
40. Question1 point(s) labor. The nurse must always consider which of the
following precautions:
Upon assessment, the nurse found the following: fundus at
2 fingerbreadths above the umbilicus, last menstrual period A. The internal exam is done only at the delivery under
(LMP) 5 months ago, fetal heartbeat (FHB) not appreciated. strict asepsis with a double set-up.
Which of the following is the most possible diagnosis of this B. The preferred manner of delivering the baby is vaginal.
condition?
C. An emergency delivery set for vaginal delivery must be
A. Hydatidiform mole made ready before examining the patient.
B. Missed abortion D. Internal exams must be done following routine
C. Pelvic inflammatory disease procedures.

D. Ectopic pregnancy Correct Answer: A. The internal exam is done only at the
delivery under strict asepsis with a double set-up.
Correct Answer: A. Hydatidiform mole
44. Question1 point(s)
41. Question1 point(s)
Which of the following signs will distinguish threatened
When a pregnant woman goes into a convulsive seizure, the abortion from imminent abortion?
MOST immediate action of the nurse to ensure the safety
of the patient is: A. Severity of bleeding.

A. Apply restraint so that the patient will not fall out of bed. B. Dilation of the cervix.

B. Put a mouth gag so that the patient will not bite her C. Nature and location of pain.
tongue and the tongue will not fall back. D. Presence of uterine contraction.
C. Position the mother on her side to allow the secretions Correct Answer: B. Dilation of the cervix
to drain from her mouth and prevent aspiration.
45. Question1 point(s)
D. Check if the woman is also having precipitate labor.
The nursing measure to relieve fetal distress due to
Correct Answer: C. Position the mother on her side to allow maternal supine hypotension is:
the secretions to drain from her mouth and prevent
aspiration. A. Place the mother in semi-Fowler's position.

42. Question1 point(s) B. Put the mother on the left side-lying position.

A gravidocardiac mother is advised to observe bed rest C. Place mother on a knee-chest position.
primarily to:
D. Any of the above.
A. Allow the fetus to achieve normal intrauterine growth.
Correct Answer: B. Put the mother on left side-lying
B. Minimize oxygen consumption which can aggravate the position.
condition of the compromised heart of the mother.
46. Question1 point(s)
C. Prevent perinatal infection.
To prevent preterm labor from progressing, drugs are
D. Reduce incidence of premature labor. usually prescribed to halt the labor. The drugs commonly
given are:
A. Magnesium sulfate and terbutaline C. Fallopian tubes

B. Prostaglandin and oxytocin D. Breast

C. Progesterone and estrogen Correct Answer: A. Cervix

D. Dexamethasone and prostaglandin

Correct Answer: A. Magnesium sulfate and terbutaline

47. Question1 point(s) 51. Question1 point(s)

In placenta previa marginalis, the placenta is found at the: Which of the following causes of infertility in the female is
primarily psychological in origin?
A. Internal cervical os partly covering the opening.
A. Vaginismus
B. External cervical os slightly covering the opening.
B. Dyspareunia
C. Lower segment of the uterus with the edges near the
internal cervical os. C. Endometriosis

D. Lower portion of the uterus completely covering the D. Impotence


cervix.
Correct Answer: A. Vaginismus
Correct Answer: C. Lower segment of the uterus with the
edges near the internal cervical os 52. Question1 point(s)

48. Question1 point(s) Before giving a repeat dose of magnesium sulfate to a pre-
eclamptic patient, the nurse should assess the patient’s
In which of the following conditions can the causative agent condition. Which of the following conditions will require the
pass through the placenta and affect the fetus in utero? nurse to temporarily suspend a repeat dose of magnesium
sulfate?
A. Gonorrhea
A. 100 cc. urine output in 4 hours
B. Rubella
B. Knee jerk reflex is (+)2
C. Candidiasis
C. Serum magnesium level is 10mEg/L.
D. Moniliasis
D. Respiratory rate of 16/min
Correct Answer: B. Rubella
Correct Answer: A. 100 cc. urine output in 4 hours
49. Question1 point(s)
53. Question1 point(s)
Which of the following can lead to infertility in adult males?
Which of the following is TRUE in Rh incompatibility?
A. German measles
A. The condition can occur if the mother is Rh(+) and the
B. Orchitis fetus is Rh(-).
C. Chickenpox B. Every pregnancy of an Rh(-) mother will result in
D. Rubella erythroblastosis fetalis.

Correct Answer: B. Orchitis C. On the first pregnancy of the Rh(-) mother, the fetus will
not be affected.
50. Question1 point(s)
D. RhoGam is given only during the first pregnancy to
Papanicolaou smear is usually done to determine cancer of: prevent incompatibility.

A. Cervix Correct Answer: C. On the first pregnancy of the Rh(-)


mother, the fetus will not be affected
B. Ovaries
54. Question1 point(s)
Which of the following are the most commonly assessed B. 25-27 weeks
findings in cystitis?
C. 28-30 weeks
A. Frequency, urgency, dehydration, nausea, chills, and
flank pain D. 38-40 weeks

B. Nocturia, frequency, urgency dysuria, hematuria, fever, Correct Answer: A. 21-24 weeks
and suprapubic pain

C. Dehydration, hypertension, dysuria, suprapubic pain, 58. Question1 point(s)


chills, and fever
A nurse in the labor room is monitoring a client with
D. High fever, chills, flank pain nausea, vomiting, dysuria, dysfunctional labor for signs of maternal or fetal
and frequency compromise. Which of the following assessment findings
Correct Answer: B. Manifestations of cystitis include, would alert the nurse to a compromise?
frequency, urgency, dysuria, hematuria nocturia, fever, and A. Coordinated uterine contractions
suprapubic pain.
B. Meconium in the amniotic fluid
55. Question1 point(s)
C. Progressive changes in the cervix
Which of the following best reflects the frequency of
reported postpartum “blues”? D. Maternal fatigue

A. Between 10% and 40% of all new mothers report some Correct Answer: B. Meconium in the amniotic fluid
form of postpartum blues.
59. Question1 point(s)
B. Between 30% and 50% of all new mothers report some
form of postpartum blues. While assessing a G2P2 client who had a normal
spontaneous vaginal delivery 30 minutes ago, the nurse
C. Between 50% and 80% of all new mothers report some notes a large amount of red vaginal bleeding. What would
form of postpartum blues. be the initial priority nursing action?

D. Between 25% and 70% of all new mothers report some A. Notify the physician.
form of postpartum blues.
B. Encourage breastfeed soon after birth.
Correct Answer: C. According to statistical reports, between
50% and 80% of all new mothers report some form of C. Monitor vital signs
postpartum blues. D. Provide fundal massage.
56. Question1 point(s) Correct Answer: D. Provide fundal massage
Which of the following conditions will lead to a small-for- 60. Question1 point(s)
gestational-age fetus due to less blood supply to the fetus?
The preferred manner of delivering the baby in a gravido-
A. Diabetes in the mother cardiac is vaginal delivery assisted by forceps under epidural
B. Maternal cardiac condition anesthesia. The main rationale for this is:

C. Premature labor A. To allow atraumatic delivery of the baby.

D. Abruptio placenta B. To allow a gradual shifting of the blood into the maternal
circulation.
Correct Answer: B. Maternal cardiac condition
C. To make the delivery effort-free and the mother does
57. Question1 point(s) not need to push with contractions.

The lower limit of viability for infants in terms of age of D. To prevent perineal laceration with the expulsion of the
gestation is: fetal head.

A. 21-24 weeks
Correct Answer: C. To make the delivery effort-free and the 64. Question1 point(s)
mother does not need to push with contractions.
The fetal heart rate is checked following rupture of the bag
61. Question1 point(s) of waters in order to:

When giving narcotic analgesics to a mother in labor, the A. Check if the fetus is suffering from head compression.
special consideration to follow is:
B. Determine if cord compression followed the rupture.

C. Determine if there is uteroplacental insufficiency.


A. The progress of labor is well established reaching the
transitional stage. D. Check if the fetal presenting part has adequately
descended following the rupture.
B. Uterine contraction is progressing well, and delivery of
the baby is imminent. Correct Answer: B. Determine if cord compression followed
the rupture.
C. Cervical dilatation has already reached at least 8 cm. and
the station is at least (+)2. 65. Question1 point(s)

D. Uterine contractions are strong and the baby will not be Upon assessment, the nurse got the following findings: two
delivered yet within the next 3 hours. (2) perineal pads highly saturated with blood within 2 hours
postpartum, PR= 80 bpm, fundus soft, and boundaries not
Correct Answer: D. Uterine contractions are strong and the well defined. The appropriate nursing diagnosis is:
baby will not be delivered yet within the next 3 hours.
A. Normal blood loss
62. Question1 point(s)
B. Blood volume deficiency
The cervical dilatation taken at 8:00 AM in a G1P0 patient
was 6 centimeters. A repeat I.E. done at 10 A. M. showed C. Inadequate tissue perfusion related to hemorrhage
that cervical dilation was 7 cm. The correct interpretation D. Hemorrhage secondary to uterine atony
of this result is:
Correct Answer: D. Hemorrhage secondary to uterine atony
A. Labor is progressing as expected.
66. Question1 point(s)
B. The latent phase of Stage 1 is prolonged.
The following are signs and symptoms of fetal distress
C. The active phase of Stage 1 is protracted. EXCEPT:
D. The duration of labor is normal. A. Fetal heart rate (FHR) decreases during a contraction
Correct Answer: C. The active phase of Stage 1 is protracted and persists even after the uterine contraction ends.

63. Question1 point(s) B. The FHR is less than 120 bpm or over 160 bpm.

Which of the following techniques during labor and delivery C. The pre-contraction FHR is 130 bpm, FHR during
can lead to uterine inversion? contraction is 118 bpm, and FHR after uterine contraction
is 126 bpm.
A. Fundal pressure applied to assist the mother in bearing
down during delivery of the fetal head. D. FHR is 160 bpm, weak and irregular.

B. Strongly tugging on the umbilical cord to deliver the Correct Answer: C. The pre-contraction FHR is 130 bpm,
placenta and hasten placental separation. FHR during contraction is 118 bpm, and FHR after uterine
contraction is 126 bpm.
C. Massaging the fundus to encourage the uterus to
contract. 67. Question1 point(s)

D. Applying light traction when delivering the placenta that If the labor period lasts only for 3 hours, the nurse should
has already detached from the uterine wall. suspect that the following conditions may occur. Select all
that apply.
Correct Answer: B. Strongly tugging on the umbilical cord to
deliver the placenta and hasten placental separation. A. Laceration of cervix
B. Laceration of perineum Correct Answer: B. From the beginning of one contraction
to the beginning of the next contraction.
C. Cranial hematoma in the fetus
71. Question1 point(s)
D. Fetal anoxia
The peak point of a uterine contraction is called the:
Correct Answer: A, B, C, & D
A. Acceleration
68. Question1 point(s)
B. Acme

C. Deceleration
The primary power involved in labor and delivery is:
D. Axiom
A. Bearing down ability of a mother.
Correct Answer: B. Acme
B. Cervical effacement and dilatation.
72. Question1 point(s)
C. Uterine contraction.
When determining the duration of a uterine contraction the
D. Valsalva technique. right technique is to time it from:
Correct Answer: C. Uterine contraction A. The beginning of one contraction to the end of the same
69. Question1 point(s) contraction.

The proper technique to monitor the intensity of a uterine B. The end of one contraction to the beginning of another
contraction is: contraction.

A. Place the palm of the hands-on the abdomen and time C. The acme point of one contraction to the acme point of
the contraction. another contraction.
B. Place the fingertips lightly on the suprapubic area and D. The beginning of one contraction to the end of another
time the contraction. contraction.

Correct Answer: A. The beginning of one contraction to the


C. Put the tip of the fingers lightly on the fundal area and end of the same contraction.
try to indent the abdominal wall at the height of the
contraction. 73. Question1 point(s)

D. Put the palm of the hands-on the fundal area and feel When the bag of water ruptures, the nurse should check the
the contraction at the fundal area. characteristic of the amniotic fluid. The normal color of
amniotic fluid is:
Correct Answer: C. Put the tip of the fingers lightly on the
fundal area and try to indent the abdominal wall at the A. Clear as water
height of the contraction.
B. Bluish
70. Question1 point(s)
C. Greenish
To monitor the frequency of the uterine contraction during
labor, the right technique is to time the contraction is: D. Yellowish

A. From the beginning of one contraction to the end of the Correct Answer: A. Clear as water
same contraction. 74. Question1 point(s)
B. From the beginning of one contraction to the beginning When the bag of waters ruptures spontaneously, the nurse
of the next contraction. should inspect the vaginal introitus for possible cord
C. From the end of one contraction to the beginning of the prolapse. If there is part of the cord that has prolapsed into
next contraction. the vaginal opening the correct nursing intervention is:

D. From the deceleration of one contraction to the acme of A. Push back the prolapsed cord into the vaginal canal.
the next contraction.
B. Place the mother in a semi fowlers position to improve
circulation.

C. Cover the prolapsed cord with sterile gauze wet with


sterile NSS and place the woman in Trendelenburg position.

D. Push back the cord into the vagina and place the woman
in Sim's position.

Correct Answer: C. Cover the prolapsed cord with sterile


gauze wet with sterile NSS and place the woman in
Trendelenburg position.

75. Question1 point(s)

The fetal heartbeat should be monitored every 15 minutes


during the 2nd stage of labor. The characteristic of a normal
fetal heart rate is:

A. The heart rate will decelerate during a contraction and


then go back to its pre-contraction rate after the
contraction.

B. The heart rate will accelerate during a contraction and


remain slightly above the pre-contraction rate at the end of
the contraction.

C. The rate should not be affected by the uterine


contraction.

D. The heart rate will decelerate at the middle of a


contraction and remain so for about a minute after the
contraction.

Correct Answer: A. The heart rate will decelerate during a


contraction and then go back to its pre-contraction rate
after the contraction.
Mechanism of labor, also known as the cardinal A. Use up-down technique with one stroke.
movements, refers to the sequencing of events involved in
posturing and positioning that allows the baby to find the B. Clean from the mons veneris to the anus.
easiest to come out of the birth canal. Arrange the following C. Use mild soap and warm water.
mechanisms of labor in the order during the course of labor
and fetal delivery. D. Paint the inner thighs going towards the perineal area.

View Answers: Correct Answer: D. Paint the inner thighs going towards the
perineal area
Engagement
5. Question1 point(s)
Descent
What are the important considerations that the nurse must
Flexion remember after the placenta is delivered? Select all that
Internal rotation apply.

Extension A. Check if the placenta is complete including the


membranes
External rotation
B. Check if the cord is long enough for the baby
Expulsion
C. Check if the umbilical cord has 3 blood vessels
2. Question1 point(s)
D. Check if the cord has a meaty portion and a shiny portion
The first thing that a nurse must ensure when the baby’s
head comes out is Correct Answer: A & C

A. The cord is intact 6. Question1 point(s)

B. No part of the cord is encircling the baby’s neck The following are correct statements about false labor,
except?
C. The cord is still attached to the placenta
A. The pain is irregular in intensity and frequency.
D. The cord is still pulsating
B. The duration of contraction progressively lengthens over
Correct Answer: B. No part of the cord is encircling the time.
baby’s neck
C. There is no bloody vaginal discharge.
3. Question1 point(s)
D. The cervix is still closed.
To ensure that the baby will breathe as soon as the head is
delivered, the nurse’s priority action is to Correct Answer: B. The duration of contraction
progressively lengthens over time
A. Slap the baby’s buttocks to make the baby cry.
7. Question1 point(s)
B. Suction the nose and mouth to remove mucous
secretions. The passageway in labor and delivery of the fetus include
the following, except?
C. Clamp the cord about 6 inches from the base.
A. Distensibility of lower uterine segment
D. Check the baby’s color to make sure it is not cyanotic.
B. Cervical dilatation and effacement
Correct Answer: B. Suction the nose and mouth to remove
mucous secretions. C. Distensibility of vaginal canal and introitus

4. Question1 point(s) D. Flexibility of the pelvis

When doing perineal care in preparation for delivery, the Correct Answer: D. Flexibility of the pelvis
nurse should observe the following, except?

8. Question1 point(s)
The normal umbilical cord is composed of: 12. Question1 point(s)

A. 2 arteries and 1 vein When the shiny portion of the placenta comes out first is
called which of the following mechanisms?
B. 2 veins and 1 artery
A. Marmets
C. 2 arteries and 2 veins
B. Ritgens
D. None of the above
C. Duncan
Correct Answer: A. 2 arteries and 1 vein
D. Schultze
9. Question1 point(s)
Correct Answer: D. Schultze
At what stage of labor and delivery does a primigravida
differ mainly from a multigravida? 13. Question1 point(s)

A. Stage 1 When the baby’s head is out, the immediate action of the
nurse is
B. Stage 2
A. Cut the umbilical cord
C. Stage 3
B. Wipe the baby’s face and suction mouth first
D. Stage 4
C. Check if there is a cord coiled around the neck
Correct Answer: A. Stage 1
D. Deliver the anterior shoulder
10. Question1 point(s)
Correct Answer: C. Check if there is a cord coiled around the
The second stage of labor begins with ___ and ends with neck.
__?
14. Question1 point(s)
A. Begins with full dilatation of cervix and ends with
delivery of placenta. When delivering the baby’s head the nurse supports the
mother’s perineum to prevent a tear. This technique is
B. Begins with true labor pains and ends with delivery of called
the baby.
A. Marmet’s technique
C. Begins with complete dilatation and effacement of cervix
and ends with delivery of the baby. B. Ritgen’s technique

D. Begins with passage of show and ends with full dilatation C. Duncan maneuver
and effacement of cervix.
D. Schultze maneuver
Correct Answer: C. Begins with complete dilatation and
effacement of cervix and ends with delivery of baby Correct Answer: B. Ritgen’s technique

11. Question1 point(s) 15. Question1 point(s)

The following are signs that the placenta has detached, The basic delivery set for normal vaginal delivery includes
except? the following instruments/articles, except?

A. Lengthening of the cord A. 2 clamps

B. Uterus becomes more globular B. Pair of scissors

C. Sudden gush of blood C. Kidney Basin

D. Mother feels like bearing down D. Retractor

Correct Answer: D. Mother feels like bearing down Correct Answer: D. Retractor
16. Question1 point(s)

As soon as the placenta is delivered, the nurse must do A. Terbutaline


which of the following actions?
B. Pitocin
A. Inspect the placenta for completeness including the
membranes. C. Magnesium sulfate

B. Place the placenta in a receptacle for disposal. D. Lidocaine

C. Label the placenta properly. Correct Answer: B. Pitocin

D. Leave the placenta in the kidney basin for the nursing 20. Question1 point(s)
aide to dispose properly. The partograph is a tool used to monitor labor. The
Correct Answer: A. Inspect the placenta for completeness maternal parameters measured/monitored are the
including the membranes. following, except?

17. Question1 point(s) A. Vital signs

In vaginal delivery done in the hospital setting, the doctor B. Fluid intake and output
routinely orders oxytocin to be given to the mother C. Uterine contraction
parenterally. The oxytocin is usually given after the placenta
has been delivered and not before because: D. Cervical dilatation

A. Oxytocin will prevent bleeding. Correct Answer: B. Fluid intake and output

B. Oxytocin can make the cervix close and thus trap the Option D: The central feature of the partogram is a graph
placenta inside. where cervical dilatation is plotted. Along the left side,
there are squares from 0 to 10, each representing 1-cm
C. Oxytocin will facilitate placental delivery. dilatation. Along the bottom of graph are numbers 0–24
D. Giving oxytocin will ensure complete delivery of the each presenting 1 h. The first stage of labor is divided into
placenta. latent and active phases. The latent phase is from 0 to 3 cm,
and it lasts up to 8 h. The active phase is from 3 to 10 cm
Correct Answer: B. Oxytocin can make the cervix close and (full cervical dilatation). The dilatation of the cervix is
thus trap the placenta inside plotted with “x.”

18. Question1 point(s) 21. Question1 point(s)

In a gravido-cardiac mother, the first 2 hours postpartum The following are natural childbirth procedures, except?
(4th stage of labor and delivery) particularly in a cesarean
section is a critical period because at this stage A. Lamaze method

A. There is a fluid shift from the placental circulation to the B. Dick-Read method
maternal circulation which can overload the compromised C. Ritgen’s maneuver
heart.
D. Psychoprophylactic method
B. The maternal heart is already weak and the mother can
die. Correct Answer: C. Ritgen’s maneuver

C. The delivery process is strenuous to the mother. 22. Question1 point(s)

D. The mother is tired and weak which can distress the The following are common causes of dysfunctional labor.
heart. Which of these can a nurse, on her own manage?

19. Question1 point(s) A. Pelvic bone contraction

This drug is usually given parenterally to enhance uterine B. Full bladder


contraction:
C. Extension rather than flexion of the head
D. Cervical rigidity Correct Answer: A. LOA

Correct Answer: B. Full bladder 27. Question1 point(s)

23. Question1 point(s) The following are types of breech presentation, except:

At what stage of labor is the mother advised to bear down? A. Footling

A. When the mother feels the pressure at the rectal area. B. Frank

B. During a uterine contraction. C. Complete

C. In between uterine contraction to prevent uterine D. Incomplete


rupture.
28. Question1 point(s)
D. Anytime the mother feels like bearing down.
When the nurse palpates the suprapubic area of the mother
Correct Answer: B. During a uterine contraction and found that the presenting part is still movable, the right
term for this observation that the fetus is
24. Question1 point(s)
A. Engaged
The normal dilatation of the cervix during the first stage of
labor in a nullipara is B. Descended

A. 1.2 cm./hr C. Floating

B. 1.5 cm./hr. D. Internal Rotation

C. 1.8 cm./hr Correct Answer: C. Floating

D. 2.0 cm./hr 29. Question1 point(s)

Correct Answer: A. 1.2 cm./hr The placenta should be delivered normally within how
many minutes after the delivery of the baby?
25. Question1 point(s)
A. 5 minutes
When the fetal head is at the level of the ischial spine, it is
said that the station of the head is B. 30 minutes

A. Station –1 C. 45 minutes

B. Station “0” D. 60 minutes

C. Station +1 Correct Answer: B. 30 minutes

D. Station +2 30. Question1 point(s)

Correct Answer: B. Station “0” When shaving a woman in preparation for cesarean section,
the area to be shaved should be from:
26. Question1 point(s)
A. Under breast to mid-thigh including the pubic area.
During an internal examination, the nurse palpated the
posterior fontanel to be at the left side of the mother at the B. The umbilicus to the mid-thigh.
upper quadrant. The interpretation is that the position of
the fetus is: C. Xiphoid process to the pubic area.

A. LOA D. Above the umbilicus to the pubic area.

B. ROP Correct Answer: A. Under breast to mid-thigh including the


pubic area.
C. LOP

D. ROA
31. Question1 point(s) Correct Answer: B. Prevent the mother from producing
antibodies against the Rh(+) antigen that she may have
During the postpartum period, the fundus of the uterus is gotten when she delivered to her Rh(+) baby
expected to go down normally about how many
centimeters per day? 35. Question1 point(s)

A. 1.0 cm To enhance milk production, a lactating mother must do the


following interventions, except:
B. 2.0 cm
A. Increase fluid intake including milk.
C. 2.5 cm
B. Eat foods that increase lactation which is called
D. 3.0 cm galactagogues.
Correct Answer: A. 1.0 cm C. Exercise adequately like aerobics.
32. Question1 point(s) D. Have adequate nutrition and rest.
The lochia on the first few days after delivery is Correct Answer: C. Exercise adequately like aerobics.
characterized as
36. Question1 point(s)
A. Pinkish with some blood clots
The nursing intervention to relieve pain in breast
B. Whitish with some mucus engorgement while the mother continues to breastfeed is
C. Reddish with some mucus A. Apply cold compress on the engorged breast.
D. Serous with some brown tinged mucus B. Apply warm compress on the engorged breast.
Correct Answer: C. Reddish with some mucus C. Massage the breast.
33. Question1 point(s) D. Apply analgesic ointment.
Lochia normally disappears after how many days Correct Answer: B. Apply warm compress on the engorged
postpartum? breast
A. 5 days 37. Question1 point(s)
B. 7-10 days A woman who delivered normally per vaginam is expected
C. 18-21 days to void within how many hours after delivery?

D. 28-30 days A. 3 hrs

Correct Answer: B. 7-10 days B. 4 hrs

34. Question1 point(s) C. 6-8 hrs

After an Rh(-) mother has delivered her Rh (+) baby, the D. 12-24 hours
mother is given RhoGam. This is done in order to: Correct Answer: C. 6-8 hrs
A. Prevent the recurrence of Rh(+) babies in future 38. Question1 point(s)
pregnancies.
To ensure adequate lactation the nurse should teach the
B. Prevent the mother from producing antibodies against mother to:
the Rh(+) antigen that she may have gotten when she
delivered to her Rh(+) baby. A. Breastfeed the baby on self-demand day and night.

C. Ensure those future pregnancies will not lead to B. Feed primarily during the day and allow the baby to
maternal illness. sleep through the night.

D. To prevent the newborn from having problems of C. Feed the baby every 3-4 hours following a strict
incompatibility when it breastfeeds. schedule.
D. Breastfeed when the breasts are engorged to ensure 42. Question1 point(s)
adequate supply.
The neonate of a mother with diabetes mellitus is prone to
Correct Answer: A. Breastfeed the baby on self-demand day developing hypoglycemia because:
and night
A. The pancreas is immature and unable to secrete the
39. Question1 point(s) needed insulin.

An appropriate nursing intervention when caring for a B. There is rapid diminution of glucose level in the baby’s
postpartum mother with thrombophlebitis is: circulating blood and his pancreas is normally secreting
insulin.
A. Encourage the mother to ambulate to relieve the pain in
the leg. C. The baby is reacting to the insulin given to the mother.

B. Instruct the mother to apply elastic bondage from the D. His kidneys are immature leading to a high tolerance for
foot going towards the knee to improve venous return flow. glucose.

C. Apply warm compress on the affected leg to relieve the Correct Answer: B. There is rapid diminution of glucose level
pain. in the baby’s circulating blood and his pancreas is normally
secreting insulin.
D. Elevate the affected leg and keep the patient on bedrest.
43. Question1 point(s)
CorrectCorrect Answer: D. Elevate the affected leg and keep
the patient on bed rest. Which of the following is an abnormal vital sign in
postpartum?
40. Question1 point(s)
A. Pulse rate between 50-60/min
The nurse should anticipate that hemorrhage related to
uterine atony may occur postnatally if this condition was B. BP diastolic increase from 80 to 95mm Hg
present during the delivery:
C. BP systolic between 100-120mm Hg
A. Excessive analgesia was given to the mother.
D. Respiratory rate of 16-20/min
B. Placental delivery occurred within thirty minutes after
the baby was born. Correct Answer: B. BP diastolic increase from 80 to 95mm
Hg
C. An episiotomy had to be done to facilitate delivery of the
head. 44. Question1 point(s)

D. The labor and delivery lasted for 12 hours. The uterine fundus right after delivery of placenta is
palpable at
Correct Answer: A. Excessive analgesia was given to the
mother. A. Level of Xiphoid process

41. Question1 point(s) B. Level of umbilicus

According to Rubin’s theory of maternal role adaptation, C. Level of symphysis pubis


the mother will go through 3 stages during the postpartum D. Midway between umbilicus and symphysis pubis
period. These stages are:
Correct Answer: B. Level of umbilicus
A. Going through, adjustment period, adaptation period
45. Question1 point(s)
B. Taking-in, taking hold and letting-go
A nurse is monitoring the amount of lochia drainage in a
C. Attachment phase, adjustment phase, adaptation phase client who is 2 hours postpartum and notes that the client
D. Taking-hold, letting-go, attachment phase has a saturated perineal pad in 1 hour. The nurse reports
the amount of lochial flow as:
Correct Answer: B. Taking-in, taking-hold and letting-go
A. Scanty

B. Light
C. Heavy A. Make the baby suck the breast regularly.

D. Excessive B. Apply ice cap on fundus.

Correct Answer: C. Heavy C. Massage the fundus vigorously for 15 minutes until
contracted.
46. Question1 point(s)
D. Give oxytocin as ordered.
In a woman who is not breastfeeding, menstruation usually
occurs after how many weeks? Correct Answer: C. Massage the fundus vigorously for 15
minutes until contracted.
A. 2-4 weeks
50. Question1 point(s)
B. 6-8 weeks
The following are nursing interventions to relieve
C. 6 months episiotomy wound pain, except:
D. 12 months A. Giving analgesic as ordered
Correct Answer: B. 6-8 weeks B. Sitz bath

C. Perineal heat
47. Question1 point(s) D. Perineal care
The following are nursing measures to stimulate lactation, Correct Answer: D. Perineal care
except:
51. Question1 point(s)
A. Frequent regular breastfeeding
Postpartum blues is said to be normal provided that the
B. Breast pumping following characteristics are present. These are
C. Breast massage A. Within 3-10 days only
D. Application of cold compress on the breast B. Woman exhibits the following symptoms- episodic
Correct Answer: D. Application of cold compress on the tearfulness, fatigue, oversensitivity, poor appetite
breast C. Maybe more severe symptoms in primipara
48. Question1 point(s) D. All of the above
When the uterus is firm and contracted after delivery but Correct Answer: D. All of the above
there is vaginal bleeding, the nurse should suspect which of
the following? 52. Question1 point(s)

A. Uterine hypercontractility The neonatal circulation differs from the fetal circulation
because
B. Uterine atony
A. The fetal lungs are non-functioning as an organ and most
C. Uterine inversion of the blood in the fetal circulation is mixed blood.
D. Laceration of soft tissues of the cervix and vagina B. The blood at the left atrium of the fetal heart is shunted
Correct Answer: D. Laceration of soft tissues of the cervix to the right atrium to facilitate its passage to the lungs.
and vagina. C. The blood in the left side of the fetal heart contains
49. Question1 point(s) oxygenated blood while the blood on the right side contains
unoxygenated blood.
The following are interventions to make the fundus
contract postpartally, except: D. None of the above.
Correct Answer: A. The fetal lungs are non-functioning as an D. Middle third of the thigh
organ and most of the blood in the fetal circulation is mixed
blood. Correct Answer: D. Middle third of the thigh

53. Question1 point(s) 56. Question1 point(s)

The normal respiration of a newborn immediately after At what APGAR score at 5 minutes after birth should
birth is characterized as: resuscitation be initiated?

A. Shallow and irregular with short periods of apnea lasting A. 1-3


not longer than 15 seconds, 30-60 breaths per minute. B. 7-8
B. 20-40 breaths per minute, abdominal breathing with C. 9-10
active use of intercostals muscles.
D. 6-7
C. 30-60 breaths per minute with apnea lasting more than
15 seconds, abdominal breathing. Correct Answer: A. 1-3

D. 30-50 breaths per minute, active use of abdominal and 57. Question1 point(s)
intercostal muscles.
Right after birth, when the skin of the baby’s trunk is pinkish
Correct Answer: A. Shallow and irregular with short periods but the soles of the feet and palm of the hands are bluish
of apnea lasting not longer than 15 seconds, 30-60 breaths this is called:
per minute.
A. Syndactyly

B. Acrocyanosis
The baby is checked at 1 minute and 5 minutes after birth
for heart and respiratory rates, muscle tone, reflexes, and C. Peripheral cyanosis
color. A baby who needs help with any of these issues is D. Cephalo-caudal cyanosis
getting constant attention during those first 5 to 10
minutes. Correct Answer: B. Acrocyanosis

54. Question1 point(s) 58. Question1 point(s)

The anterior fontanelle is characterized as: The minimum birth weight for full-term babies to be
considered normal is:
A. 3-4 cm anteroposterior diameter and 2-3 cm transverse
diameter, diamond shape. A. 2,000 gms

B. 2-3 cm anteroposterior diameter and 3-4 cm transverse B. 1,500 gms


diameter and diamond shape.
C. 2,500 gms
C. 2-3 cm in both anteroposterior and transverse diameter
and diamond shape. D. 3,000 gms

D. None of the above. Correct Answer: C. 2,500 gms

Correct Answer: A. 3-4 cm anteroposterior diameter and 2- 59. Question1 point(s)


3 cm transverse diameter, diamond shape.
This procedure is done to prevent ophthalmia neonatorum
55. Question1 point(s) is:

The ideal site for vitamin K injection in the newborn is: A. Marmet’s technique

A. Right upper arm B. Ophthalmic wash

B. Left upper arm C. Ritgen’s method

C. Either right or left buttocks D. Crede’s method

Correct Answer: D. Crede’s method


60. Question1 point(s) C. The mother uses mixed feeding faithfully.

Which of the following characteristics will distinguish a D. The mother breastfeeds regularly until 1 year with no
postmature neonate at birth? supplemental feedings.

A. Plenty of lanugo and vernix caseosa. Correct Answer: B. The mother breastfeeds exclusively and
regularly during the first 6 months without giving
B. Lanugo mainly on the shoulders and vernix in the skin supplemental feedings.
folds.
64. Question1 point(s)
C. Pinkish skin with good turgor.
The intrauterine device prevents pregnancy by the ffg
D. Almost leather-like, dry, cracked skin, negligible vernix mechanism, except:
caseosa.
A. Endometrium inflames.
Correct Answer: D. Almost leather-like, dry, cracked skin,
negligible vernix caseosa B. Fundus contracts to expel uterine contents.

61. Question1 point(s) C. Copper embedded in the IUD can kill the sperms.

What would be the appropriate first nursing action when D. Sperms will be barred from entering the fallopian tubes.
caring for a 20-year old G1P0 woman at 39 weeks gestation
who is in active labor and for whom an assessment reveals Correct Answer: D. Sperms will be barred from entering the
mild variable fetal heart rate deceleration fallopian tubes.

A. Notify the physician 65. Question1 point(s)

B. Prepare the client for immediate delivery Oral contraceptive pills are of different types. Which type is
most appropriate for mothers who are breastfeeding?
C. Readjust the fetal monitor
A. Estrogen-only
D. Change the maternal position
B. Progesterone only
Correct Answer: D. Change the maternal position
C. Mixed type- estrogen and progesterone
62. Question1 point(s)
D. 21-day pills mixed type
In basal body temperature (BBT) technique, the sign that
ovulation has occurred is an elevation of body temperature Correct Answer: B. Progesterone only
by 66. Question1 point(s)
A. 1.0-1.4 degrees centigrade The natural family planning method called Standard Days
B. 0.2-0.4 degrees centigrade (SDM), is the latest type and easy to use method. However,
it is a method applicable only to women with regular
C. 2.0-4.0 degrees centigrade menstrual cycles between how many days?

D. 1.0-4.0 degrees centigrade A. 21-26 days

Correct Answer: B. 0.2-0.4 degrees centigrade B. 26-32 days

63. Question1 point(s) C. 28-30 days

Lactation Amenorrhea Method(LAM) can be an effective D. 24- 36 days


method of natural birth control if
Correct Answer: B. 26-32 days
A. The mother breastfeeds mainly at night time when
ovulation could possibly occur.

B. The mother breastfeeds exclusively and regularly during


the first 6 months without giving supplemental feedings.
67. Question1 point(s) D. Undergo a complete medical check-up to rule out any
debilitating disease
Nurse Lesley is conducting health teachings to a group of
first-time mothers. Which of the following are signs of Correct Answer: A, B, & D
ovulation? Select all that apply.
71. Question1 point(s)
A. Mittelschmerz
In the sympto-thermal method, the parameters being
B. Spinnbarkeit monitored to determine if the woman is fertile or infertile
are:
C. Thin watery cervical mucus
A. Temperature, cervical mucus, cervical consistency
D. Elevated body temperature of 4.0 degrees centigrade
B. Release of ovum, temperature, and vagina
Correct Answer: A, B, & C
C. Temperature and wetness
68. Question1 point(s)
D. Temperature, endometrial secretion, mucus
The following methods of artificial birth control works as a
barrier device, except Correct Answer: A. Temperature, cervical mucus, cervical
consistency
A. Condom
72. Question1 point(s)
B. Cervical cap
The following are important considerations to teach the
C. Cervical Diaphragm woman who is on a low dose (mini-pill) oral contraceptive
D. Intrauterine device (IUD) except:

Correct Answer: D. Intrauterine device (IUD) A. The pill must be taken every day at the same time.

69. Question1 point(s) B. If the woman fails to take a pill in one day, she must take
2 pills for added protection.
Which of the following is a true statement about normal
ovulation? C. If the woman fails to take a pill in one day, she needs to
take another temporary method until she has consumed
A. It occurs on the 14th day of every cycle. the whole pack.

B. It may occur between 14-16 days before next D. If she is breastfeeding, she should discontinue using
menstruation. mini-pill and use the progestin-only type.

C. Every menstrual period is always preceded by ovulation. Correct Answer: B. If the woman fails to take a pill in one
day, she must take 2 pills for added protection.
D. The most fertile period of a woman is 2 days after
ovulation. 73. Question1 point(s)

Correct Answer: B. It may occur between 14-16 days before To determine if the cause of infertility is a blockage of the
next menstruation. fallopian tubes, the test to be done is

70. Question1 point(s) A. Huhner’s test

If a couple would like to enhance their fertility, the following B. Postcoital test
means can be done. Select all that apply.
C. Rubin’s test
A. Monitor the basal body temperature of the woman
every day to determine peak period of fertility. D. None of the above

B. Have adequate rest and nutrition Correct Answer: C. Rubin’s test

C. Have sexual contact only during the dry period of the


woman
74. Question1 point(s)

Infertility can be attributed to male causes such as the


following except:

A. Cryptorchidism

B. Orchitis

C. Sperm count of about 20 million per milliliter

D. Premature ejaculation

Correct Answer: C. Sperm count of about 20 million per


milliliter

75. Question1 point(s)

Spinnbarkeit is an indicator of ovulation which is


characteized as:

A. Thin watery mucus which can be stretched into a long


strand about 10 cm.

B. Thick mucus that is detached from the cervix during


ovulation.

C. Thin mucus that is yellowish in color with fishy odor.

D. Thick mucus vaginal discharge is influenced by high


levels of estrogen.

Correct Answer: A. Thin watery mucus which can be


stretched into a long strand about 10 cm

You might also like